You are on page 1of 70

CHAPTER 5

1. RIZAL SURETY & INSURANCE COMPANY, petitioner,


vs.
COURT OF APPEALS and TRANSWORLD KNITTING MILLS, INC., G.R. No. 112360

FACTS:

Rizal Surety & Insurance Company issued a fire insurance policy in favor of Transworld Knitting Mills, Inc.
The subject policy stated that Rizal Surety is “responsible in case of loss whilst contained and/or stored during the
currency of this Policy in the premises occupied by them forming part of the buildings situated within own
Compound xxx.” The policy also described therein the four-span building covered by the same.

On Jan. 12, 1981, fire broke out in the compound, razing the middle portion of its four-span building and
partly gutting the left and right sections thereof. A two-storey building (behind said four-span building) was also
destroyed by the fire.

ISSUE:

Whether or not Rizal Surety is liable for loss of the two-storey building considering that the fire insurance
policy sued upon covered only the contents of the four-span building

RULING:

Yes. Both the trial court and the CA found that the so-called “annex” as not an annex building but an
integral and inseparable part of the four-span building described in the policy and consequently, the machines and
spare parts stored therein were covered by the fire insurance in dispute.

So also, considering that the two-storey building aforementioned was already existing when subject fire
insurance policy contract was entered into on Jan. 12, 1981, having been constructed some time in 1978,
petitioner should have specifically excluded the said two-storey building from the coverage of the fire insurance if
minded to exclude the same but if did not, and instead, went on to provide that such fire insurance policy covers
the products, raw materials and supplies stored within the premises of Transworld which was an integral part of
the four-span building occupied by Transworld, knowing fully well the existence of such building adjoining and
intercommunicating with the right section of the four-span building. Also, in case of doubt in the stipulation as to
the coverage of the fire insurance policy, under Art. 1377 of the New Civil Code, the doubt should be resolved
against the Rizal Surety, whose layer or managers drafted the fire insurance policy contract under scrutiny.

In Landicho vs. Government Service Insurance System, the Court ruled that “the terms in an insurance
policy, which are ambiguous, equivocal or uncertain x x x are to be construed strictly and most strongly against the
insurer, and liberally in favor of the insured so as to effect the dominant purpose of indemnity or payment to the
insured, especially where forfeiture is involved, and the reason for this is that the insured usually has no voice in
the selection or arrangement of the words employed and that the language of the contract is selected with great
care and deliberation by experts and legal advisers employed by, and acting exclusively in the interest of, the
insurance company.”
2. MALAYAN INSURANCE CORPORATION, petitioner,
vs.
THE HON. COURT OF APPEALS and TKC MARKETING CORPORATION, G.R. No. 119599

FACTS:

TKC Marketing imported 3,000 metric tons of soya from Brazil to Manila. It was insured by Malayan at the
value of almost 20 million pesos. The vessel, however, was stranded on South Africa because of a lawsuit regarding
the possession of the soya. TKC consulted Malayan on recovery of the amount, but the latter claimed that it wasn’t
covered by the policy. The soya was sold in Africa for Php 10 million, but TKC wanted Malayan to shoulder the
remaining value of 10 million as well.

Petitioner filed suit due to Malayan’s reticence to pay. Malayan claimed that arrest by civil authorities
wasn’t covered by the policy. The trial court ruled in TKC’s favor with damages to boot. The appellate court
affirmed the decision under the reason that clause 12 of the policy regarding an excepted risk due to arrest by civil
authorities was deleted by Section 1.1 of the Institute War Clauses which covered ordinary arrests by civil
authorities. Failure of the cargo to arrive was also covered by the Theft, Pilferage, and Non-delivery Clause of the
contract. Hence this petition.

ISSUE:
1. WON the arrest of the vessel was a risk covered under the subject insurance policies.
2. WON the insurance policies must strictly construed against the insurer.

RULING:

1. Yes. Section 12 or the "Free from Capture & Seizure Clause" states: "Warranted free of capture,
seizure, arrest, restraint or detainment, and the consequences thereof or of any attempt thereat… Should Clause
12 be deleted, the relevant current institute war clauses shall be deemed to form part of this insurance.”

This was really replaced by the subsection 1.1 of section 1 of Institute War Clauses (Cargo) which included
“the risks excluded from the standard form of English Marine Policy by the clause warranted free of capture,
seizure, arrest, restraint or detainment, and the consequences thereof of hostilities or warlike operations, whether
there be a declaration of war or not.” The petitioner’s claim that the Institute War Clauses can be operative in case
of hostilities or warlike operations on account of its heading "Institute War Clauses" is not tenable. It reiterated the
CA’s stand that “its interpretation in recent years to include seizure or detention by civil authorities seems
consistent with the general purposes of the clause.” This interpretation was regardless of the fact whether the
arrest was in war or by civil authorities.

2. Yes. Indemnity and liability insurance policies are construed in accordance with the general rule of
resolving any ambiguity therein in favor of the insured, where the contract or policy is prepared by the insurer. A
contract of insurance, being a contract of adhesion, means that any ambiguity should be resolved against the
insurer.
3. PHILIP S. YU, Petitioner,
vs.
HON. COURT OF APPEALS, G.R. No. 154115

FACTS:

On 15 March 1994, Viveca Lim Yu (private respondent) brought against her husband, Philip Sy Yu
(petitioner), an action for legal separation and dissolution of conjugal partnership on the grounds of marital
infidelity and physical abuse. The case was filed before the RTC of Pasig. During trial, private respondent Lim Yu
moved for the issuance of a subpoena duces tecum and ad testificandum, to certain officers of Insular Life
Assurance Co. Ltd. to compel production of the insurance policy and application of a person suspected to be
petitioner’s illegitimate child. The trial court denied the motion, It ruled that the insurance contract is inadmissible
evidence in view of Circular Letter No. 11-2000, issued by the Insurance Commission which presumably prevents
insurance companies/agents from divulging confidential and privileged information pertaining to insurance
policies. It added that the production of the application and insurance contract would violate Article 280 of the
Civil Code and Section 5 of the Civil Registry Law, both of which prohibit the unauthorized identification of the
parents of an illegitimate child. Private respondent sought reconsideration of the Order, but the motion was
denied by the trial court. On appeal to the CA, private respondent was merely seeking the production of the
insurance application and contract, and was not yet offering the same as part of her evidence. Thus, it declared
that petitioner’s objection to the admission of the documents was premature, and the trial court’s pronouncement
that the documents are inadmissible, precipitate

ISSUE:

Whether or not an insurance policy and its corresponding application form can be admitted as evidence to
prove a party’s extra-marital affairs in an action for legal separation

RULING:

The insurance application and the insurance policy were yet to be presented in court, much less formally
offered before it. In fact, private respondent was merely asking for the issuance of subpoena duces tecum and
subpoena ad testificandum when the trial court issued the assailed Order. Even assuming that the documents
would eventually be declared inadmissible, the trial court was not then in a position to make a declaration to that
effect at that point. Thus, it barred the production of the subject documents prior to the assessment of its
probable worth. As observed by petitioners, the assailed Order was not a mere ruling on the admissibility of
evidence; it was, more importantly, a ruling affecting the proper conduct of trial. Excess of jurisdiction refers to any
act which although falling within the general powers of the judge is not authorized and is consequently void with
respect to the particular case because the conditions under which he was only authorized to exercise his general
power in that case did not exist and therefore, the judicial power was not legally exercised.Thus, in declaring that
the documents are irrelevant

and inadmissible even before they were formally offered, much less presented before it, the trial court acted in
excess of its discretion.
4. DOMINGO GARCIA and THE PHILIPPINE NATIONAL BANK, plaintiffs-appellees,
vs.
THE HONGKONG FIRE & MARINE INSURANCE CO., LTD., G.R. No. 20341

FACTS:

Garcia had his merchandise insured by Hongkong Fire and Marine Insurance Co. The insurance company however
made a mistake and issued a policy covering the building where the merchandise was stored. (The building was not
owned by Garcia). The policy was written in English, of which Garcia was ignorant, so he could not have noticed the
error of the insurance company. Said policy was later on assigned by Garcia to PNB to secure a loan. PNB
acknowledged receipt of said policy, referring to it as a policy covering the merchandise. > The insurance company
made the necessary endorsements to PNB. The building which housed the merchandise was later razed by fire.
The insurance company refused to pay due to the fact that the policy indicates insurance on the building and not
on the merchandise.

ISSUE:

Whether or not Garcia can collect.

RULING:

Yes. The defense of the insurer is purely technical. The mistake was obviously on the part of the insurer when it
issued a wrong policy. It cannot deny such allegation due to the fact that it even confirmed with PNB the nature of
said policy when it was endorsed. Garcia could not have noticed the mistake due to his ignorance of the English
language.
5. TAN CHUCO, plaintiff, appellant-appellee,
vs.
YORKSHIRE FIRE AND LIFE INSURANCE COMPANY, G.R. No. L-5069

FACTS:

Tan Chuco files a claim under an open fire insurance policy forthe alleged loss by fire of certain stock of goods
insured by Yorkshire. CFI: Evidence did not sustain Yorkshire’s allegation that TanChuco or his agents had
intentionally and fraudulently set thebuilding on fire. But was of the opinion that the Tan Chuco failedto establish
the value of the goods he allegeswere destroyed by the fire

ISSUE:

Whether or not Tan Chuco may claim under the fireinsurance policy?

RULING:

No. We think that the action of the trial court in rejecting the proof offered by Tan Chuco as to the amount of the
loss must besustained. The contract of fire insurance being a contract of indemnity, TanChuco is only entitled to
recover the amount of actual loss sustainedby him.
6. PHILAMCARE HEALTH SYSTEMS, INC., petitioner,
vs.
COURT OF APPEALS and JULITA TRINOS, G.R. No. 125678

FACTS:

Ernani Trinos applied for a health care coverage with Philam. He answered no to a question asking if he or
his family members were treated to heart trouble, asthma, diabetes, etc. The application was approved for 1 year.
He was also given hospitalization benefits and out-patient benefits. After the period expired, he was given an
expanded coverage for Php 75,000. During the period, he suffered from heart attack and was confined at MMC.
The wife tried to claim the benefits but the petitioner denied it saying that he concealed his medical history by
answering no to the aforementioned question. She had to pay for the hospital bills amounting to 76,000. Her
husband subsequently passed away. She filed a case in the trial court for the collection of the amount plus
damages. She was awarded 76,000 for the bills and 40,000 for damages. The CA affirmed but deleted awards for
damages. Hence, this appeal.

ISSUE:

WON a health care agreement is not an insurance contract; hence the “incontestability clause” under the
Insurance Code does not apply.

RULING:

No. Petitioner claimed that it granted benefits only when the insured is alive during the one-year
duration. It contended that there was no indemnification unlike in insurance contracts. It supported this claim by
saying that it is a health maintenance organization covered by the DOH and not the Insurance Commission. Lastly,
it claimed that the Incontestability clause didn’t apply because two-year and not one-year effectivity periods were
required. Section 2 (1) of the Insurance Code defines a contract of insurance as “an agreement whereby one
undertakes for a consideration to indemnify another against loss, damage or liability arising from an unknown or
contingent event.”
Section 3 states: every person has an insurable interest in the life and health:(1) of himself, of his spouse and of his
children.

In this case, the husband’s health was the insurable interest. The health care agreement was in the nature
of non-life insurance, which is primarily a contract of indemnity. The provider must pay for the medical expenses
resulting from sickness or injury. While petitioner contended that the husband concealed materialfact of his
sickness, the contract stated that: “that any physician is, by these presents, expressly authorized to disclose or give
testimony at anytime relative to any information acquired by him in his professional capacity upon any question
affecting the eligibility for health care coverage of the Proposed Members.” This meant that the petitioners
required him to sign authorization to furnish reports about his medical condition. The contract also authorized
Philam to inquire directly to his medical history.

Hence, the contention of concealment isn’t valid.


7. LEE BOG & COMPANY, plaintiff-appellee,
vs.
THE HANOVER FIRE INSURANCE COMPANY OF THE CITY OF NEW YORK, ET AL., defendants-appellants.
REPUBLIC OF THE PHILIPPINES, ET AL., G.R. No. L-10305

FACTS:

The insurance covered "stock of rice and palay (loose and/or sacks), the property of the assured or held by
him in trust, on commission or on joint account, with others and/or for which he is... responsible in case of loss",
while contained during the currency of the policies in the building of the assured in Binalonan, Pangasinan,
otherwise known as the Binalonan, Pangasinan Rice Mill. There was a common "simple loss payable clause" in
favor of the Bureau of Commerce... in all the policies issued by defendants-appellants, except Policy No. 1016373,
issued by the Hanover Fire Insurance Company, which also contained a "simple loss payable clause" but in favor of
the People's Surety & Insurance Co., Inc. Said clause provides that "loss, if... any, under this policy, is payable to the
Bureau of Commerce, Manila, as its interest may appear, subject to the terms, conditions, clauses, and warranties
of this policy."

In this instant appeal, it is argued that the lower court erred in considering the claims on the bonded palay
belonging to depositors separately and independently from the claim on the unbonded palay belonging to the
appellee because the policies sued upon were concurrent and... each and all of them covered, in their entirety,
inseparably and indivisibly, the stock of rice and palay kept in the insured's warehouse, whether belonging to the
insured or to its depositors

ISSUE:

Under the second assignment of error, appellants contend that appellee has failed to establish its loss; that the
claims were for about three times the actual loss and therefore fraudulent; that appellee employed fraudulent
means and devices to obtain undue benefits under the... policies by combining and commingling with sacks of rice
and palay approximately the same quantity of rice bran and/or rice husk; and that appellee had presented false
supporting declarations.

RULING:

Naturally, numerical precision may not be expected, because those estimates were based merely on a physical
observation of the big pile existing before the fire. It is sufficient that they show little discrepancy with the... figures
recorded in the books of the appellee.

The mathematical computations of witnesses Filomeno and Magpili are "rough estimates" and therefore some
allowance for such technical factors as "staggering," "shrinkage" and "angle of repose" should be duly taken into
account; and where said estimates do not show too wide a... difference, there would be no justification in
discrediting appellee's claims.

We also overrule the contention that the appellee used fraudulent means or devices to obtain benefits under the
policies. The conclusion that, because the samples of the debris taken from the warehouse after the fire consisted
of darak and rice husks, these must have... been contained in the sacks stored in the warehouse, is untenable. In
the first place, it is not unusual to find such debris because their unburned material formed the protective lining of
the sacks of palay. Secondly, the samples were taken only from the sides of the pile and... not from its core.
Thirdly, considering (as appellee argues) the size of the pile, 1,411.84 cubic meters, and the time it took the fire to
consume the mass of palay, the samples taken are too insignificant to be representative. Lastly, the motive for
such alleged fraud is... missing. Appellee company was having a thriving business at the time of the fire.

8. PACIFIC TIMBER EXPORT CORPORATION, petitioner,


vs.
THE HONORABLE COURT OF APPEALS and WORKMEN'S INSURANCE COMPANY, INC., G.R. No. L-38613

FACTS:

March 19, l963: Pacific Timber secured temporary insurance from Workmen's Insurance Company, Inc. for
its exportation of 1,250,000 board feet of Philippine Lauan and Apitong logs to be shipped from the Diapitan Bay,
Quezon Province to Tokyo, Japan. Workmen's issued Cover Note insuring the cargo "Subject to the Terms and
Conditions of the Workmen's Insurance Company, Inc."

On April 2, 1963 regular marine cargo policies were issued for a total of 1,195.498 bd. ft. Due to the bad weather
some of the logs were lost during loading operations. 45 pieces of logs were salvaged, but 30 pieces were lost.
Pacific informed Workmen's who refused stating that the logs covered in the 2 marine policies were received in
good order at the point of destination and that the cover note was null and void upon the issuance of the Marine
Policies

ISSUE:

W/N the cover note is valid despite the absence of premium payment upon it

RULING:

Yes. It was not necessary to ask for payment of the premium on the Cover Note , for the loss insured
against having already occurred, the more practical procedure is simply to deduct the premium from the amount
due on the Cover Note. Had all the logs been lost during the loading operations, but after the issuance of the Cover
Note, liability on the note would have already arisen even before payment of premium cover note as a
"binder"supported by the doctrine that where a policy is delivered without requiring payment of the premium, the
presumption is that a credit was intended and policy is valid it sent its adjuster to investigate and assess the loss to
determine if petitioner was guilty of delay in communicating the loss but there was none Section 84. Delay in the
presentation to an insurer of notice or proof of loss is waived if caused by any act of his or if he omits to take
objection promptly and specifically upon that ground.
9. DEVELOPMENT INSURANCE CORPORATION, petitioner,
vs.
INTERMEDIATE APPELLATE COURT, and PHILIPPINE UNION REALTY DEVELOPMENT CORPORATION, G.R. No.
71360

FACTS:

A fire occurred in the building of Philippine Union. It sued for recovery of damages from the petitioner on the basis
of an insurance contract between them. The petitioner failed to answer on time despite the numerous extensions
it asked for. It was declared in default by the trial court. A judgment of default was subsequently rendered on the
strength of the evidence given by the private respondent, which was allowed damages. The petitioner moved to
lift the order of default. Its motion was denied. It went to the appellate court, which affirmed the decision of the
trial court. Hence this appeal.

ISSUE:

Was Philippine Union required to jointly indemnify the building?

RULING:

No. The policy insured the private respondent's building against fire for P2,500,000.00.

The petitioner argued that the respondent must share the difference between that amount and the face
value of the policy and the loss sustained for 5.8 million under Condition 17 of the policy.

The building was insured at P2,500,000.00 by agreement of the insurer and the insured. The agreement is
known as an open policy and is subject to the express condition that: “In the event of loss, whether total or partial,
it is understood that the amount of the loss shall be subject to appraisal and the liability of the company, if
established, shall be limited to the actual loss, subject to the applicable terms, conditions, warranties
and clauses of this Policy, and in no case shall exceed the amount of the policy.” Section 60 of
the Insurance Code defines an open policy is one in which the value of the thing insured is not agreed upon but is
left to be ascertained in case of loss." This means that the actual loss, as determined, will represent the total
indemnity due the insured from the insurer except only that the total indemnity shall not exceed the face value of
the policy.

The actual loss has been ascertained in this case. Hence, applying the open policy clause as expressly
agreed upon, the private respondent is entitled to indemnity in the total amount of P508,867.00. The refusal of its
vice-president to receive the private respondent's complaint was the first indication of the petitioner's intention to
prolong this case and postpone the discharge of its obligation to the private respondent under this agreement.
They still evaded payment for 5 years.
10. FIELDMEN'S INSURANCE CO., INC., petitioner,
vs.
MERCEDES VARGAS VDA. DE SONGCO, ET AL. and COURT OF APPEALS, G.R. No. L-24833

FACTS:

The facts as found by respondent Court of Appeals, binding upon us, follow: “This is a peculiar case. Federico
Songco of Floridablanca, Pampanga, a man of scant education, being only a first grader . . ., owned a private
jeepney with Plate No. 41-289 for the year 1960. On September 15, 1960, as such private vehicle owner, he was
induced by FIELDMEN’S Insurance Company Pampanga agent Benjamin Sambat to apply for a Common Carrier’s
Liability Insurance Policy covering his motor vehicle .. Upon paying an annual premium of P16.50, defendant
FIELDMEN’S Insurance Company Inc. issued on September 19, 1960, Common Carriers Accident Insurance Policy
No. 45-HO-4254 the duration of which will be for one (1) year, effective September 15, 1960 to September 15,
1961. On September 22, 1961, the defendant company, upon payment of the corresponding premium, renewed
the policy by extending the coverage from October 15, 1961 to October 15, 1962. This time Federico Songco’s
private jeepney carried Plate No. J-68136- Pampanga – 1961 . On October 29, 1961, during the effectivity of the
renewed policy, the insured vehicle while being driven by Rodolfo Songco, a duly licensed driver and son of
Federico (the vehicle owner) collided with a car in the municipality of Calumpit, province of Bulacan, as a result of
which mishap Federico Songco (father) and Rodolfo Songco (son) died, Carlos Songco (another son), the latter’s
wife, Angelita Songco, and a family friend by the name of Jose Manuel sustained physical injuries of varying
degrees.”

ISSUE:

WON THE INSURER SHOULD PAY FOR THE PROCEEDS OF THE INSURANCE.

RULING:

YES. COMMERCIAL LAWS; INSURANCE CONTRACTS; COMMON CARRIER LIABILITY INSURANCE; INSURER WHO
REPRESENTS INSURABILITY OF VEHICLE ESTOPPED FROM DENYING LIABILITY THEREON. — After petitioner
FIELDMEN’S Insurance Co., Inc., had led the insured Federico Songco to believe that he could qualify under the
common carrier liability insurance policy, and to enter into contract of insurance paying the premiums due, it could
not, thereafter, in any litigation arising out of such representation, be permitted to change its stand to the
detriment of the heirs of the insured. As estoppel is primarily based on the doctrine of good faith and the
avoidance of harm that will befall the innocent party due to its injurious reliance, the failure to apply it in this case
would result in a gross travesty of justice.
11. FE DE JOYA LANDICHO, in her own behalf and as judicial guardian of her minor children, RAFAEL J.
LANDICHO and MA. LOURDES EUGENIA LANDICHO,plaintiffs-appellees,
vs.
GOVERNMENT SERVICE INSURANCE SYSTEM,defendant-appellant. G.R. No. L-28866

FACTS:

  =$ 
   
&   
& 
2 
   


4>!!
#7
%+
?
7+
&


;#;
#2
)
%=$
 @A!!



#=$



&



&35
4


'
B&
)

-
%
)

        ( 

&
#
&+


&35
35
4

&'


C6
C6

=$


""#"D
;
 >DE
&
(
'


B
  +   
  
 %&  
  
  
  

 
%  $  'F
 F 
    
   #( 
  $

&
#
  =$ 
   
&   
& 
2 
   
  


4>!!
#7
%+
?
7+
&


;#;
#2
)
%=$
 @A!!



#=$



&



&35
4


'
B&
)

-
%
)
(

&
#
&+


&35
35
4

&'


C6
C6

=$


""#"D
;
 >DE
&
(
'


B
  +   
  
 %&  
  
  
  

 
%  $  'F
 F 
    
   #( 
  $

&
#
On June 1, 1964, the GSIS issued in favor of Flaviano Landicho, a civil engineer of the Bureau of Public Works,
stationed at Mamburao, Mindoro Occidental, optional additional life insurance policy No. OG-136107 in the sum of
P7,900. Landicho had filed an application, by filing and signing a printed form of the GSIS on the basis of which the
policy was issued. Paragraph 7 of said application States: 7. xxx I hereby agree as follows: xxx c. That this
application serves as a letter of authority to the Collecting Officer of our Office thru the GSIS to deduct from my
salary the monthly premium in the amount of P33.36, beginning the month of May, 1964, and every month
thereafter until notice of its discontinuance shall have beenreceived from the System; . d. That the failure to
deduct from my salary the month premiums shall not make the policy lapse, however, the premium account shall
be considered as indebtedness which, I bind myself to pay the System; . e. That my policy shall be made effective
on the first day of the month next following the month the first premium is paid; provided, that it is not more
ninety (90) days before or after the date of the medical examination, was conducted if required." While still an
employee of the Bureau of Public Works, Mr. Landicho died in an airplane crash on June 29, 1966. Mrs. Landicho,
in her own behalf and that of her co-plaintiffs and minor children, Rafael J. and Maria Lourdes Eugenia, filed with
the GSIS a claim for P15,800, as the double indemnity due under policy No. OG-136107. GSIS denied the claim,
upon the ground that the policy had never been in force because, pursuant to subdivision (e) of the above-quoted
paragraph 7 of the application, the policy "shall be ... effective on the first day of the month next following the
month the first premium is paid," and no premium had ever been paid on said policy. The Lower Court decided in
favor of the petitioner. GSIS appealed to the Supreme Court.

ISSUE:

WON the insurance policy in question has ever been in force, not a single premium having been paid thereon.

RULING:

Lower Court decision is sustained. (T)he language, of subdivisions (c), (d) and (e) is such as to create an ambiguity
that should be resolved against the party responsible therefor — defendant GSIS, as the party who prepared and
furnished the application form — and in favor of the party misled thereby, the insured employee. Indeed, our Civil
Code provides: The interpretation of obscure words or stipulations in a contract shall not favor the party who
caused the obscurity.

12. MALAYAN INSURANCE CO., INC., Petitioner,


vs.
REGIS BROKERAGE CORP., G.R. No. 172156

FACTS:

June 7, 1981: Malayan insurance co., inc. (MICO) issued to Coronacion Pinca, Fire Insurance Policy for her property
effective July 22, 1981, until July 22, 1982. On October 15,1981, MICO allegedly cancelled the policy for non-
payment, of the premium and sent the corresponding notice to Pinca. On December 24, 1981, payment of the
premium for Pinca was received by Domingo Adora, agent of MICO. On January 15, 1982, Adora remitted this
payment to MICO,together with other payments. On January 18, 1982, Pinca’s property was completely burned.
On February 5, 1982, Pinca’s payment was returned by MICO to Adora on the ground that her policy had been
cancelled earlier but Adora refused to accept it and instead demanded for payment. Under Section 416 of the
Insurance Code, the period for appeal is thirty days from notice of the decision of the Insurance Commission. The
petitioner filed its motion for reconsideration on April 25, 1981, or fifteen days such notice, and the reglementary
period began to run again after June 13, 1981, date of its receipt of notice of the denial of the said motion for
reconsideration. As the herein petition was filed on July 2, 1981, or nineteen days later, there is no question that it
is tardy by four days. Insurance Commission, favored Pinca MICO appealed

ISSUE:

W/N MICO should be liable because its agent Adora was authorized to receive it

RULING:

YES. petition is DENIED

SEC. 77. An insurer is entitled to payment of the premium as soon as the thing is exposed to the peril insured
against. Notwithstanding any agreement to the contrary, no policy or contract of insurance issued by an insurance
company is valid and binding unless and until the premium thereof has been paid, except in the case of a life or an
industrial life policy whenever the grace period provision applies.
SEC. 306. xxx xxx xxx

Any insurance company which delivers to an insurance agant or insurance broker a policy or contract of insurance
shall be demmed to have authorized such agent or broker to receive on its behalf payment of any premium which
is due on such policy or contract of insurance at the time of its issuance or delivery or which becomes due thereon.
Payment to an agent having authority to receive or collect payment is equivalent to payment to the principal
himself; such payment is complete when the money delivered is into the agent’s hands and is a discharge of the
indebtedness owing to the principal.

13. SAN MIGUEL BREWERY, ETC., plaintiff-appellee,


vs.
LAW UNION AND ROCK INSURANCE CO., (LTD.) ET AL., G.R. No. L-14300

FACTS:

In the contract of mortgage, the owner P.D. Dunn had agreed, at his own expense, to insure the
mortgaged property for its full value and to indorse the policies in such manner as to authorize the Brewery
Company to receive the proceeds in case of loss and to retain such part thereof as might be necessary to satisfy
the remainder then due upon the mortgage debt. Instead, however, of effecting the insurance himself Dunn
authorized and requested the Brewery Company to procure insurance on the property in the amount of P15,000 at
Dunn's expense.
San Miguel insured the property only as mortgagee.

Dunn sold the propert to Henry Harding. The insurance was not assigned by Dunn to Harding. When it
was destroyed by fire, the two companies settled with San Miguelto the extent of the mortgage credit. RTC:
Absolved the 2 companies from the difference. Henry Harding is not entitled to the difference between the
mortgage credit and the face value of the policies. Henry Harding appealed.

ISSUE:

1. W/N San Miguel has insurable interest as mortgagor only to the extent of the mortgage credit - YES
2. W/N Harding has insurable interest as owner - NO

RULING:

The Insurance Act: a change of interest in any part of a thing insured unaccompanied by a corresponding
change of interest in the insurance, suspends the insurance to an equivalent extent, until the interest in the thing
and the interest in the insurance are vested in the same person
section 55: the mere transfer of a thing insured does not transfer the policy, but suspends it until the same person
becomes the owner of both the policy and the thing insured

Undoubtedly these policies of insurance might have been so framed as to have been "payable to the San
Miguel Brewery, mortgagee, as its interest may appear, remainder to whomsoever, during the continuance of the
risk, may become the owner of the interest insured." (Sec 54, Act No. 2427.) Such a clause would have proved an
intention to insure the entire interest in the property, not merely the insurable interest of the San Miguel Brewery,
and would have shown exactly to whom the money, in case of loss, should be paid. But the policies are not so
written.
The blame for the situation thus created rests, however, with the Brewery rather than with the insurance
companies, and there is nothing in the record to indicate that the insurance companies were requested to write
insurance upon the insurable interest of the owner or intended to make themselves liable to that extent

If by inadvertence, accident, or mistake the terms of the contract were not fully set forth in the policy, the
parties are entitled to have it reformed. But to justify the reformation of a contract, the proof must be of the most
satisfactory character, and it must clearly appear that the contract failed to express the real agreement between
the parties In the case now before us the proof is entirely insufficient to authorize reformation.

14. SUN INSURANCE OFFICE, LTD., petitioner,


vs.
COURT OF APPEALS and EMILIO TAN, G.R. No. 89741

FACTS:

On August 15, 1983, herein private respondent Emilio Tan took from herein petitioner a P300,000.00 property
insurance policy to cover his interest in the electrical supply store of his brother housed in a building in Iloilo City.
Four (4) days after the issuance of the policy, the building was burned including the insured store. On August 20,
1983, Tan filed his claim for fire loss with petitioner, but on February 29, 1984, petitioner wrote Tan denying the
latter’s claim. On April 3, 1984, Tan wrote petitioner, seeking reconsideration of the denial of his claim. On
September 3, 1985, Tan’s counsel wrote the Insurer inquiring about the status of his April 3, 1984 request for
reconsideration. Petitioner answered the letter on October 11, 1985, advising Tan’s counsel that the Insurer’s
denial of Tan’s claim remained unchanged, enclosing copies of petitioners’ letters of February 29, 1984 and May
17, 1985 (response to petition for reconsideration). On November 20, 1985, Tan filed Civil Case No. 16817 with the
Regional Trial Court of Iloilo, Branch 27 but petitioner filed a motion to dismiss on the alleged ground that the
action had already prescribed. Said motion was denied in an order dated November 3, 1987; and petitioner’s
motion for reconsideration was also denied in an order dated January 14, 1988.

ISSUE:

WON THE ACTION HAS PRESCRIBED.

RULING:

YES. While it is a cardinal principle of insurance law that a policy or contract of insurance is to be construed
liberally in favor of the insured and strictly against the insurer company, yet, contracts of insurance, like other
contracts, are to be construed according to the sense and meaning of the terms which the parties themselves have
used. If such terms are clear and unambiguous, they must be taken and understood in their plain, ordinary and
popular sense (Pacific Banking Corp. v. Court of Appeals, 168 SCRA 1[1988]).

15. SAURA IMPORT & EXPORT CO., INC., plaintiff-appellant,


vs.
PHILIPPINE INTERNATIONAL SURETY CO., INC., and PHILIPPINE NATIONAL BANK, G.R. No. L-15184

FACTS:

Saura Import & Export Co Inc., mortgaged to the Phil. National Bank, a parcel of land. The mortgage was
amended to guarantee an increased amount, bringing the total mortgaged debt to P37,000 On the land mortgage
is a building owned by Saura Import & Export Co Inc. which was insured with Philippine International Surety
(Insurer) even before the mortgage contract so it was required to endorse to mortgagee PNB

October 15, 1954: Barely 13 days after the issuance of the fire insurance policy, the insurer cancelled it.
Notice of the cancellation was given to PNB (mortgagee). But Saura (insured) was not informed. April 6, 1955: The
building and all its contents worth P40,685.69 were burned so Saura filed a claim with the Insurer and mortgagee
Bank RTC: dismissed

ISSUE:
W/N Philippine International Surety should be held liable for the claim because notice to only the mortgagee is
not substantial

RULING:

YES. Appealed from is hereby reversed. Philippine International Surety Co., Inc., to pay Saura Import & Export Co.,
Inc., P29,000. It was the primary duty of Philippine International Surety to notify the insured, but it did not

If a mortgage or lien exists against the property insured, and the policy contains a clause stating that loss, if any,
shall be payable to such mortgagee or the holder of such lien as interest may appear, notice of cancellation to the
mortgagee or lienholder alone is ineffective as a cancellation of the policy to the owner of the property.

liability attached principally the insurance company, for its failure to give notice of the cancellation of the policy to
Saura

it is unnecessary to discuss the errors assigned against appellee bank

CHAPTER 6

16. FIELDMEN'S INSURANCE CO., INC., petitioner,


vs.
MERCEDES VARGAS VDA. DE SONGCO, ET AL. and COURT OF APPEALS, respondents.

FACTS:
The facts as found by respondent Court of Appeals, binding upon us, follow: “This is a peculiar case. Federico
Songco of Floridablanca, Pampanga, a man of scant education, being only a first grader . . ., owned a private
jeepney with Plate No. 41-289 for the year 1960. On September 15, 1960, as such private vehicle owner, he was
induced by FIELDMEN’S Insurance Company Pampanga agent Benjamin Sambat to apply for a Common Carrier’s
Liability Insurance Policy covering his motor vehicle. Upon paying an annual premium of P16.50, defendant
FIELDMEN’S Insurance Company Inc. issued on September 19, 1960, Common Carriers Accident Insurance Policy
No. 45-HO-4254 .the duration of which will be for one (1) year, effective September 15, 1960 to September 15,
1961. On September 22, 1961, the defendant company, upon payment of the corresponding premium, renewed
the policy by extending the coverage from October 15, 1961 to October 15, 1962. This time Federico Songco’s
private jeepney carried Plate No. J-68136- Pampanga – 1961 . On October 29, 1961, during the effectivity of the
renewed policy, the insured vehicle while being driven by Rodolfo Songco, a duly licensed driver and son of
Federico (the vehicle owner) collided with a car in the municipality of Calumpit, province of Bulacan, as a result of
which mishap Federico Songco (father) and Rodolfo Songco (son) died, Carlos Songco (another son), the latter’s
wife, Angelita Songco, and a family friend by the name of Jose Manuel sustained physical injuries of varying
degrees.”

ISSUE:
WON THE INSURER SHOULD PAY FOR THE PROCEEDS OF THE INSURANCE.

RULING:

Yes. COMMERCIAL LAWS; INSURANCE CONTRACTS; COMMON CARRIER LIABILITY INSURANCE; INSURER WHO
REPRESENTS INSURABILITY OF VEHICLE ESTOPPED FROM DENYING LIABILITY THEREON. — After petitioner
FIELDMEN’S Insurance Co., Inc., had led the insured Federico Songco to believe that he could qualify under the
common carrier liability insurance policy, and to enter into contract of insurance paying the premiums due, it could
not, thereafter, in any litigation arising out of such representation, be permitted to change its stand to the
detriment of the heirs of the insured. As estoppel is primarily based on the doctrine of good faith and the
avoidance of harm that will befall the innocent party due to its injurious reliance, the failure to apply it in this case
would result in a gross travesty of justice.

17. ORIENTAL ASSURANCE CORPORATION, petitioner,


vs.
COURT OF APPEALS AND PANAMA SAW MILL CO., INC., respondents.

FACTS:
Panama Sawmill shipped 1208 pieces of apitog logs to Manila and insured the logs with Oriental for the value of
Php 1 million. Two barges were loaded with 610 and 598 logs. At sea, typhoons ravaged one of the barges,
resulting in the loss of 497 of 598 of the logs. The Insurance contract provided for indemnity under the following
conditions:
Warranted that this Insurance is against TOTAL LOSS ONLY. Subject to the following clauses:
— Civil Code Article 1250 Waiver clause
— Typhoon warranty clause
— Omnibus clause.
Oriental didn’t give an indemnity because there wasn’t total loss of the shipment. The sawmill filed a civil case
against Oriental and the court ordered it to pay 410,000 as value for the missing logs. The CA affirmed the lower
court judgment but reduced the legal interest. Hence this appeal by Oriental.

ISSUE:
Whether or not Oriental Assurance can be held liable under its marine insurance policy based on the theory of a
divisible contract of insurance and, consequently, a constructive total loss

RULING:

No. Perla v CA- The terms of the contract constitute the measure of the insurer liability and compliance therewith
is a condition precedent to the insured's right to recovery from the insurer.

“Whether a contract is entire or severable is a question of intention to be determined by the language employed
by the parties. The policy in question shows that the subject matter insured was the entire shipment of 2,000 cubic
meters of apitong logs. The fact that the logs were loaded on two different barges did not make the contract
several and divisible as to the items insured. The logs on the two barges were not separately valued or separately
insured. Only one premium was paid for the entire shipment, making for only one cause or consideration. The
insurance contract must, therefore, be considered indivisible.”

18. VIRGINIA CALANOC, petitioner,


vs.
COURT OF APPEALS and THE PHILIPPINE AMERICAN LIFE INSURANCE CO., respondents.

FACTS:
This suit involves the collection of P2,000 representing the value of a supplemental policy covering accidental
death which was secured by one Melencio Basilio from the Philippine American Life Insurance Company.

Melencio Basilio was a watchman of the Manila Auto Supply located at the corner of Avenida Rizal and Zurbaran.
He secured a life insurance policy from the Philippine American Life Insurance Company in the amount of P2,000 to
which was attached a supplementary contract covering death by accident. On January 25, 1951, he died of a
gunshot wound on the occasion of a robbery committed in the house of Atty. Ojeda at the corner of Oroquieta and
Zurbaan streets. Virginia Calanoc, the widow, was paid the sum of P2,000, face value of the policy, but when she
demanded the payment of the additional sum of P2,000 representing the value of the supplemental policy, the
company refused alleging, as main defense, that the deceased died because he was murdered by a person who
took part in the commission of the robbery and while making an arrest as an officer of the law which contingencies
were expressly excluded in the contract and have the effect of exempting the company from liability.

ISSUE:
Whether the Philippine American Insurance is liable for the supplemental contract

RULING:
Yes. The circumstance that he was a mere watchman and had no duty to heed the call of Atty. Ojeda should not be
taken as a capricious desire on his part to expose his life to danger considering the fact that the place he was in
duty-bound to guard was only a block away. In volunteering to extend help under the situation, he might have
thought, rightly or wrongly, that to know the truth was in the interest of his employer it being a matter that affects
the security of the neighborhood. No doubt there was some risk coming to him in pursuing that errand, but that
risk always existed it being inherent in the position he was holding. He cannot therefore be blamed solely for doing
what he believed was in keeping with his duty as a watchman and as a citizen. And he cannot be considered as
making an arrest as an officer of the law, as contended, simply because he went with the traffic policeman, for
certainly he did not go there for that purpose nor was he asked to do so by the policeman.

19. EMILIO TAN, JUANITO TAN, ALBERTO TAN and ARTURO TAN, petitioners,
vs.
THE COURT OF APPEALS and THE PHILIPPINE AMERICAN LIFE INSURANCE COMPANY, respondents.

FACTS:
Tan Lee Siong, father of the petitioners, applied for life insurance in the amount of P 80,000.00 with Philamlife. It
was approved. Tan Lee Siong died of hepatoma. Petitioners then filed a claim for the proceeds. The company
denied petitioners' claim and rescinded the policy by reason of the alleged misrepresentation and concealment of
material facts. The premiums paid on the policy were refunded. The petitioners filed a complaint in the Insurance
Commission. The latter dismissed the complaint.

The Court of Appeals dismissed ' the petitioners' appeal from the Insurance Commissioner's decision for lack of
merit. Hence, this petition.

ISSUE:
WON Philam didn’t have the right to rescind the contract of insurance as rescission must allegedly be done during
the lifetime of the insured within two years and prior to the commencement of action.
RULING:

RULING:
No, the policy was in force for a period of only one year and five months. Considering that the insured died before
the two-year period had lapsed, respondent company is not, therefore, barred from proving that the policy is void
ab initio by reason of the insured's fraudulent concealment or misrepresentation. The "incontestability clause"
added by the second paragraph of Section 48 is in force for two years. After this, the defenses of concealment or
misrepresentation no longer lie.

The petitioners argue that no evidence was presented to show that the medical terms were explained in a
layman's language to the insured. They also argue that no evidence was presented by respondent company to
show that the questions appearing in Part II of the application for insurance were asked, explained to and
understood by the deceased so as to prove concealment on his part. This couldn’t be accepted because the insured
signed the form. He affirmed the correctness of all the entries. The company records show that the deceased was
examined by Dr. Victoriano Lim and was found to be diabetic and hypertensive. He was also found to have suffered
from hepatoma. Because of the concealment made by the deceased, the company was thus misled into accepting
the risk and approving his application as medically fit.

20. RAFAEL (REX) VERENDIA, petitioner,


vs.
COURT OF APPEALS and FIDELITY & SURETY CO. OF THE PHILIPPINES, respondents.

FACTS:
Rafael (Rex) Verendia's residential building was insured with Fidelity and Surety Insurance Company, Country
Bankers Insurance and Development Insurance with Monte de Piedad & Savings Bank as beneficiary. December 28,
1980 early morning: the building was completely destroyed by fire Fidelity refused the claim stating that there was
a misrepresentation since the lessee was not Roberto Garcia but Marcelo Garcia trial court: favored Fidelity CA:
reversed

ISSUE:
W/N there was false declaration which would forfeit his benefits under Section 13 of the policy

RULING:
Yes. Section 13 thereof which is expressed in terms that are clear and unambiguous, that all benefits under the
policy shall be forfeited "If the claim be in any respect fraudulent, or if any false declaration be made or used in
support thereof, or if any fraudulent means or devises are used by the Insured or anyone acting in his behalf to
obtain any benefit under the policy"

Robert Garcia then executed an affidavit before the National Intelligence and Security Authority (NISA) to the
effect that he was not the lessee of Verendia's house and that his signature on the contract of lease was a
complete forgery.
Worse yet, by presenting a false lease contract, Verendia, reprehensibly disregarded the principle that insurance
contracts are uberrimae fidae and demand the most abundant good faith

21. YU PANG CHENG alias YU PANG CHING, petitioner,


vs.
THE COURT OF APPEALS, ET AL., respondents.
FACTS:
Yu Pang Eng submitted application for insurance consisting of the medical declaration made by him to the medical
examiner and the report. Yu then paid the premium in the sum of P591.70.

The insured, in his application for insurance, said “no” to ever having stomach disease, cancer, and fainting-spells.
He also claimed to not have consulted a physician regarding such diseases.
After submitting the form, he entered the hospital where he complained of dizziness, anemia, abdominal pains and
tarry stools. He was found to have peptic ulcer. The insured entered another hospital for medical treatment but he
died of "infiltrating medullary carcinoma, Grade 4, advanced cardiac and of lesser curvature, stomach metastases
spleen." Yu Pang Cheng aimed to collect P10,000.00 on life of one Yu Pang Eng from an insurance company. The
company set up the defense that the insured was guilty of misrepresentation and concealment of material facts.
They subsequently refused to give the indemnity.

ISSUE:
Whether or not the insured is guilty of concealment of some facts material to the risk insured that consequently
avoids the policy.

RULING:
Yes. The first confinement took place from January 29, 1950 to February 11, while his application was submitted on
September 5, 1950. When he gave his answers to the policy, he concealed the ailment of which he was treated in
the hospital. The negative answers given by the insured regarding his previous ailment deprived defendant of the
opportunity to make the necessary inquiry as to the nature of his past illness so that as it may form its estimate
relative to the approval of his application. Had defendant been given such opportunity, the company would
probably had never consented to the issuance of the policy in question. In fact, according to the death certificate,
the insured’s death may have direct connection with his previous illness.

22. PARIS-MANILA PERFUME CO., also known as PARIS-MANILA PERFUMERY CO., plaintiff-appellee,
vs.
PHOENIX ASSURANCE CO., LTD., defendant-appellant.
FACTS:
 May 22, 1924: A fire insurance policy was issued by Phoenix Assurance Company, Limited to Messrs. Paris-
Manila Perfumery Co. (Peter Johnson, Prop.) for P13,000 
 also insured with other insurance companies for P1,200 and P5,000 respectively
 July 4, 1924: The Perfumery was burned unknown of the cause totalling a loss of P38.025.56
 Phoenix refused to pay nor to appoint an arbitrator stating that the policy did not cover any loss or
damage occasioned by explosion and stating that the claim was fraudulent
 RTC: ordered Phoenix to pay P13,000
 Phoenix appealed
 The insurance policy contains: 
Unless otherwise expressly stated in the policy the insurance does not cover 
(h)    Loss or damage occasioned by the explosion; but loss or damage by explosion of gas for illuminating or
domestic purposes in a building in which gas is not generated and which does not form a part of any gas works, will
be deemed to be loss by fire within the meaning of this policy.

ISSUE:
W/N Phoenix should be liable for the loss because there was no explosion which is an exemption from the policy

RULING:
Yes. If it be a fact that the fire resulted from an explosion that fact, if proven, would be a complete defense, the
burden of the proof of that fact is upon the defendant, and upon that point, there is a failure of proof lower court
found as a fact that there was no fraud in the insurance, and that the value of the property destroyed by the fire
was more than the amount of the insurance.

23. PRUDENTIAL GUARANTEE and ASSURANCE INC., petitioner,


vs.
TRANS-ASIA SHIPPING LINES, INC., Respondent.
FACTS:
Trans-Asia is the owner of the vessel M/V Asia Korea. In consideration of payment of premiums, PRUDENTIAL
Guarantee, insured M/V Asia Korea for loss/damage of the hull and machinery arising from perils inter alia of fire
and explosion for the sum of ₱ 40 million, beginning from the period of July 1, 1993 up to July 1, 1994. On October
25, 1993, while the policy was in force, a fire broke out while [M/V Asia-Korea was] undergoing repairs at the port
of Cebu. On October 26, 1993 Trans-Asia filed its notice of claim for damaged sustained by the vessel evidenced by
a letter/formal claim. TRANS-ASIA reserved its right to subsequently notify PRUDENTIAL to the full amount of the
claim upon final survey and determination by Adjuster Richard Hogg (Phil.) of the damaged sustained by reason of
fire. TRANS-ASIA executed a document denominated “Loan and Trust Receipt”, a portion of which states that
“Received from Prudential Guarantee and Assurance, Inc., the sum of ₱ 3,000,000.00, as a loan without internet
under Policy No. MH 93/1353, repayable only in the event and to the extent that any net recovery is made by
TRANS-ASIA Shipping Corporation, from any person or persons, corporation or corporation, or other parties, on
account of loss by any casualty for which they may be liable occasioned by the 25 October 1993 Fire on Board.

ISSUE:
Whether or not TRANS-ASIA breached the warranty stated in the insurance policy, thus absolving PRUDENTIAL
from paying TRANS-ASIA.

RULING:
No. As found by the Court of Appeals and as supported by the records, Bureau Veritas is a classification society
recognized in the marine industry. As it is undisputed that TRANS-ASIA was properly classed at the time the
contract of insurance was entered into, thus, it becomes incumbent upon PRUDENTIAL to show evidence that the
status of TRANS-ASIA as being properly CLASSED by Bureau Veritas had shifted in violation of the warranty.
Unfortunately, PRUDENTIAL failed to support the allegation.

It is generally accepted that a warranty is a statement or promise set forth in the policy, or by reference
incorporated therein, the untruth or non-fulfillment of which in any respect, and without reference to whether the
insurer was in fact prejudiced by such untruth or non-fulfillment, renders the policy voidable by the insurer; For
the breach of warranty to avoid a policy, the same must be duly shown by the party alleging the same.—We are
not unmindful of the clear language of Sec. 74 of the Insurance Code which provides that, “the violation of a
material warranty, or other material provision of a policy on the part of either party thereto, entitles the other to
rescind.” It is generally accepted that “[a] warranty is a statement or promise set forth in the policy, or by
reference incorporated therein, the untruth or non-fulfillment of which in any respect, and without reference to
whether the insurer was in fact prejudiced by such untruth or non-fulfillment, renders the policy voidable by the
insurer.” However, it is similarly indubitable that for the breach of a warranty to avoid a policy, the same must be
duly shown by the party alleging the same. We cannot sustain an allegation that is unfounded. Consequently,
PRUDENTIAL, not having shown that TRANS-ASIA breached the warranty condition, CLASSED AND CLASS
MAINTAINED, it remains that TRANSASIA must be allowed to recover its rightful claims on the policy.

24. TAN CHAY HENG, plaintiff-appellee,


vs.
THE WEST COAST LIFE INSURANCE COMPANY, defendant-appellant.
FACTS:
Tan Chay applied for a life insurance policy of for the sum of P10,000 where he was the sole beneficiary. The
company approved this. The policy was issued upon the payment by Tan Ceang of the first year's premium worth
P936. The company agreed to pay the beneficiay the amount of the policy upon the receipt of the proofs of the
death of the insured while the policy was in force. Without any premium due or unpaid, Ceang died. Tan Chay
plaintiff submitted the proofs of the death of Tan Ceang with a claim for the payment. The company refused to
pay.

The company alleged that Tan Ceang obtained the policy by means of deceit to the effect that the medical
certificate had false statements about his health. They also claimed that he didn’t pay the premium. The court
ruled for Tan Chay and commanded the company to pay 10,000 pesos.

ISSUE:
WON Section 47 of the Insurance Code applies to this case.

RULING:
No. The plaintiff contends that section 47 of the Insurance Act should be applied, and that when so applied, the
company is barred and estopped to plead the matters alleged in its special defense. That section states: Whenever
a right to rescind a contract of insurance is given to the insurer by any provision of this chapter, such right must be
exercised previous to the commencement of an action on the contract. The defendant contends that section 47
does not apply to this special defense. If in legal effect defendant's special defense is in the nature of an act to
rescind "a contract of insurance," then such right must be exercised prior to an action enforce the contract.

In the instant case, the defendant does not seek to have the alleged insurance contract rescinded. It only denies
that it ever made any contract of insurance on the life of Tan Ceang or that any such a contract ever existed. If the
defendant never made or entered into the contract in question, there is no contract to rescind, and, hence, section
47 doesn’t apply. As stated, an action to rescind a contract is founded upon and presupposes the existence of the
contract which is sought to be rescinded.

25. FINMAN GENERAL ASSURANCE CORPORATION, petitioner,


vs.
THE HONORABLE COURT OF APPEALS and JULIA SURPOSA, respondents.

FACTS:
[P]etitioner filed this petition alleging grove abuse of discretion on the part of the appellate court in applying the
principle of “expresso unius exclusio alterius” in a personal accident insurance policy since death resulting from
murder and/or assault are impliedly excluded in said insurance policy considering that the cause of death of the
insured was not accidental but rather a deliberate and intentional act of the assailant in killing the former as
indicated by the location of the lone stab wound on the insured. Therefore, said death was committed with
deliberate intent which, by the very nature of a personal accident insurance policy, cannot be indemnified.

ISSUE:
Whether or not death petitioner is correct that results from assault or murder deemed are not included in the
terms “accident” and “accidental”.

RULING:
NO. Petition for certiorari with restraining order and preliminary injunction was denied for lack of merit. The terms
“accident” and “accidental” as used in insurance contracts have not acquired any technical meaning, and are
construed by the courts in their ordinary and common acceptation. Thus, the terms have been taken to mean that
which happen by chance or fortuitously, without intention and design, and which is unexpected, unusual, and
unforeseen. An accident is an event that takes place without one’s foresight or expectation — an event that
proceeds from an unknown cause, or is an unusual effect of a known cause and, therefore, not expected.

[I]t is well settled that contracts of insurance are to be construed liberally in favor of the insured and strictly
against the insurer. Thus ambiguity in the words of an insurance contract should be interpreted in favor of its
beneficiary.

26. DOMINGO GARCIA and THE PHILIPPINE NATIONAL BANK, plaintiffs-appellees,


vs.
THE HONGKONG FIRE & MARINE INSURANCE CO., LTD., defendant-appellant.

FACTS:
 Garcia had his merchandise insured by Hongkong Fire and Marine Insurance Co.
 The insurance company however made a mistake and issued a policy covering the building where the
merchandise was stored. (The building was not owned by Garcia)
 The policy was written in English, of which Garcia was ignorant, so he could not have noticed the error of
the insurance company.
 Said policy was later on assigned by Garcia to PNB to secure a loan. PNB acknowledged receipt of said
policy, referring to it as a policy covering the merchandise.
 The insurance company made the necessary endorsements to PNB.
 The building which housed the merchandise was later razed by fire. The insurance company refused to
pay due to the fact that the policy indicates insurance on the building and not on the merchandise.

ISSUE:
Whether or not Garcia can collect.

RULING:
Yes. The defense of the insurer is purely technical. The mistake was obviously on the part of the insurer when it
issued a wrong policy. It cannot deny such allegation due to the fact that it even confirmed with PNB the nature of
said policy when it was endorsed. Garcia could not have noticed the mistake due to his ignorance of the English
language.

27. NG GAN ZEE, plaintiff-appellee,


vs.
ASIAN CRUSADER LIFE ASSURANCE CORPORATION, defendant-appellant.

FACTS:
insurance on his life for the sum of P20,000.00, with his wife, appellee Ng Gan Zee as beneficiary. On the same
date, Asian Crusader, upon receipt of the required premium from the insured, approved the application and issued
the corresponding policy. Kwong Nam died of cancer of the liver with metastasis. All premiums had been paid at
the time of his death. Ng Gan Zee presented a claim for payment of the face value of the policy. On the same date,
she submitted the required proof of death of the insured. Appellant denied the claim on the ground that the
answers given by the insured to the questions in his application for life insurance were untrue.mAppellee brought
the matter to the attention of the Insurance Commissioner. The latter, after conducting an investigation, wrote the
appellant that he had found no material concealment on the part of the insured and that, therefore, appellee
should be paid the full face value of the policy. The company refused to settle its obligation. Appellant alleged that
the insured was guilty of misrepresentation when he answered "No" to the following question appearing in the
application for life insurance- Has any life insurance company ever refused your application for insurance or for
reinstatement of a lapsed policy or offered you a policy different from that applied for? If, so, name company and
date.

ISSUE:
WON Asian Crusader was deceived into entering the contract or in accepting the risk at the rate of premium
agreed upon because of insured's representation?

RULING:
No. Sec. 27. Such party a contract of insurance must communicate to the other, in good faith, all facts within his
knowledge which are material to the contract, and which the other has not the means of ascertaining, and as to
which he makes no warranty.

"Concealment exists where the assured had knowledge of a fact material to the risk, and honesty, good faith, and
fair dealing requires that he should communicate it to the assurer, but he designedly and intentionally withholds
the same." It has also been held "that the concealment must, in the absence of inquiries, be not only material, but
fraudulent, or the fact must have been intentionally withheld." Fraudulent intent on the part of the insured must
be established to entitle the insurer to rescind the contract. And as correctly observed by the lower court,
"misrepresentation as a defense of the insurer to avoid liability is an 'affirmative' defense. The duty to establish
such a defense by satisfactory and convincing evidence rests upon the defendant. The evidence before the Court
does not clearly and satisfactorily establish that defense." It bears emphasis that Kwong Nam had informed the
appellant's medical examiner of the tumor. His statement that said tumor was "associated with ulcer of the
stomach" should be construed as an expression made in good faith of his belief as to the nature of his ailment and
operation.
28. MRS. HENRY E. HARDING, and her husband, plaintiffs-appellees,
vs.
COMMERCIAL UNION ASSURANCE COMPANY, defendant-appellant.

FACTS:
Smith Bell insured Mrs. Hardings’ Studebaker car for a premium of Php 150. It was insured for Php 3,000, the value
of the car. The car was destroyed by fire. Mrs. Harding furnished the defendant the proofs of her loss, but the
company didn’t pay. Evidence showed that Hermanos sold the automobile to Canson for P3,200. Canson then sold
the car to Harding for Php 1,500. The car was then sold for P2,000. It was then resold to Harding. He gave the car
to his wife; Mrs. Henry E. Harding as a present. The automobile was repaired and repainted at the Luneta Garage
at P900. The company averred that they gave false information, particularly that on the price of the vehicle and the
ownership of the car. Hence, they aimed to declare the policy void. The trial court found that there was no fraud.
This was an action by plaintiffs to recover from defendant the sum of P3,000 and interest, alleged to be due under
the terms of a policy of insurance. The trial court gave plaintiffs judgment for the amount demanded, with interest
and costs, and from that decision the defendant appealed.

ISSUE:
Was the valuation of the car for P3000 done fraudulently, thereby making the policy void?

RULING:
No. The policy stated that “That during the period above set forth and during any period for which the company
may agree to renew this policy the company will subject to the exception and conditions contained herein or
endorsed hereon indemnify the insured against loss of or damage to any motor car described in the schedule by
whatever cause such loss or damage may be occasioned and will further indemnify the insured up to the value of
the car or P3,000 whichever is the greater against any claim at common law made by any person for loss of life or
for accidental bodily injury or damage to property caused by the said motor car including law costs payable in
connection with such claim when incurred with the consent of the company.” Defendant contends that the
statement regarding the cost of the automobile was a warranty, that the statement was false, and that, therefore,
the policy never attached to the risk.
29. NG GAN ZEE, plaintiff-appellee,
vs.
ASIAN CRUSADER LIFE ASSURANCE CORPORATION, defendant-appellant.

FACTS:
insurance on his life for the sum of P20,000.00, with his wife, appellee Ng Gan Zee as beneficiary. On the same
date, Asian Crusader, upon receipt of the required premium from the insured, approved the application and issued
the corresponding policy. Kwong Nam died of cancer of the liver with metastasis. All premiums had been paid at
the time of his death. Ng Gan Zee presented a claim for payment of the face value of the policy. On the same date,
she submitted the required proof of death of the insured. Appellant denied the claim on the ground that the
answers given by the insured to the questions in his application for life insurance were untrue.mAppellee brought
the matter to the attention of the Insurance Commissioner. The latter, after conducting an investigation, wrote the
appellant that he had found no material concealment on the part of the insured and that, therefore, appellee
should be paid the full face value of the policy. The company refused to settle its obligation. Appellant alleged that
the insured was guilty of misrepresentation when he answered "No" to the following question appearing in the
application for life insurance- Has any life insurance company ever refused your application for insurance or for
reinstatement of a lapsed policy or offered you a policy different from that applied for? If, so, name company and
date.

ISSUE:
WON Asian Crusader was deceived into entering the contract or in accepting the risk at the rate of premium
agreed upon because of insured's representation?

RULING:
No. Sec. 27. Such party a contract of insurance must communicate to the other, in good faith, all facts within his
knowledge which are material to the contract, and which the other has not the means of ascertaining, and as to
which he makes no warranty.

"Concealment exists where the assured had knowledge of a fact material to the risk, and honesty, good faith, and
fair dealing requires that he should communicate it to the assurer, but he designedly and intentionally withholds
the same." It has also been held "that the concealment must, in the absence of inquiries, be not only material, but
fraudulent, or the fact must have been intentionally withheld." Fraudulent intent on the part of the insured must
be established to entitle the insurer to rescind the contract. And as correctly observed by the lower court,
"misrepresentation as a defense of the insurer to avoid liability is an 'affirmative' defense. The duty to establish
such a defense by satisfactory and convincing evidence rests upon the defendant. The evidence before the Court
does not clearly and satisfactorily establish that defense." It bears emphasis that Kwong Nam had informed the
appellant's medical examiner of the tumor. His statement that said tumor was "associated with ulcer of the
stomach" should be construed as an expression made in good faith of his belief as to the nature of his ailment and
operation.
30. MALAYAN INSURANCE CORPORATION, petitioner,
vs.
THE HON. COURT OF APPEALS and TKC MARKETING CORPORATION, respondents.

FACTS:
TKC Marketing imported 3,000 metric tons of soya from Brazil to Manila. It was insured by Malayan at the value of
almost 20 million pesos. The vessel, however, was stranded on South Africa because of a lawsuit regarding the
possession of the soya. TKC consulted Malayan on recovery of the amount, but the latter claimed that it wasn’t
covered by the policy. The soya was sold in Africa for Php 10 million, but TKC wanted Malayan to shoulder the
remaining value of 10 million as well. Petitioner filed suit due to Malayan’s reticence to pay. Malayan claimed that
arrest by civil authorities wasn’t covered by the policy. The trial court ruled in TKC’s favor with damages to boot.
The appellate court affirmed the decision under the reason that clause 12 of the policy regarding an excepted risk
due to arrest by civil authorities was deleted by Section 1.1 of the Institute War Clauses which covered ordinary
arrests by civil authorities. Failure of the cargo to arrive was also covered by the Theft, Pilferage, and Non-delivery
Clause of the contract. Hence this petition.

ISSUE:
1. WON the arrest of the vessel was a risk covered under the subject insurance policies.
2. WON the insurance policies must strictly construed against the insurer.

RULING:
1. Section 12 or the "Free from Capture & Seizure Clause" states: "Warranted free of capture, seizure,
arrest, restraint or detainment, and the consequences thereof or of any attempt thereat… Should Clause
12 be deleted, the relevant current institute war clauses shall be deemed to form part of this insurance.”
This was really replaced by the subsection 1.1 of section 1 of Institute War Clauses (Cargo) which included
“the risks excluded from the standard form of English Marine Policy by the clause warranted free of
capture, seizure, arrest, restraint or detainment, and the consequences thereof of hostilities or warlike
operations, whether there be a declaration of war or not.” The petitioner’s claim that the Institute War
Clauses can be operative in case of hostilities or warlike operations on account of its heading "Institute
War Clauses" is not tenable. It reiterated the CA’s stand that “its interpretation in recent years to include
seizure or detention by civil authorities seems consistent with the general purposes of the clause.” This
interpretation was regardless of the fact whether the arrest was in war or by civil authorities.
2. 2. Indemnity and liability insurance policies are construed in accordance with the general rule of resolving
any ambiguity therein in favor of the insured, where the contract or policy is prepared by the insurer. A
contract of insurance, being a contract of adhesion, means that any ambiguity should be resolved against
the insurer.
CHAPTER 7

31. MALAYAN INSURANCE CO., INC. (MICO), petitioner,


vs.
GREGORIA CRUZ ARNALDO, in her capacity as the INSURANCE COMMISSIONER, and CORONACION

PINCA, respondents

FACTS:

On June 7, 1981, the petitioner (hereinafter called (MICO) issued to the private respondent, Coronacion Pinca, Fire
Insurance Policy No. F-001-17212 on her property for the amount of P14,000.00 effective July 22, 1981, until July
22, 1982. On October 15,1981, MICO allegedly cancelled the policy for non-payment, of the premium and sent the
corresponding notice to Pinca.

On December 24, 1981, payment of the premium for Pinca was received by Domingo Adora, agent of MICO. On
January 15, 1982, Adora remitted this payment to MICO, together with other payments. On January 18, 1982,
Pinca's property was completely burned.

ISSUE:

Whether there was a valid insurance contract at the time of the loss.

RULING:

Yes. A valid cancellation must, therefore, require concurrence of the following conditions:
(1) There must be prior notice of cancellation to the insured;
(2) The notice must be based on the occurrence, after the effective date of the policy, of one or more of the
grounds mentioned;
(3) The notice must be
(a) in writing,
(b) mailed, or delivered to the named insured,
(c) at the address shown in the policy;
(4) It must state
(a) which of the grounds mentioned in Section 64 is relied upon and
(b) that upon written request of the insured, the insurer will furnish the facts on which the cancellation is based.

MICO's claims it cancelled the policy in question on October 15, 1981, for non-payment of premium. To support
this assertion, it presented one of its employees, who testified that "the original of the endorsement and credit
memo" presumably meaning the alleged cancellation — "were sent the assured by mail through our mailing
section" However, there is no proof that the notice, assuming it complied with the other requisites mentioned
above, was actually mailed to and received by Pinca. We also look askance at the alleged cancellation, of which the
insured and MICO's agent himself had no knowledge, and the curious fact that although Pinca's payment was
remitted to MICO's by its agent on January 15, 1982, MICO sought to return it to Adora only on February 5, 1982,
after it presumably had learned of the occurrence of the loss insured against on January 18, 1982. These
circumstances make the motives of the petitioner highly suspect, to say the least, and cast serious doubts upon its
candor and bona fides.

32. BONIFACIO BROS., INC., ET AL., plaintiffs-appellants,


vs.
ENRIQUE MORA, ET AL., defendants-appellees.

FACTS:

Enrique Mora, owner of an Olds Mobile sedan model 1956, mortgaged the same to H.S. Reyes Inc., with the
condition that the former would insure the automobile with the latter as beneficiary. The automobile was insured
on June 23, 1959 with the State Bonding & Insurance Co., Inc. and a motor car insurance was issued to Enrique
Mora

During the effectivity of the insurance contract the car met an accident. Mora, without knowledge of the H. S.
Reyes Inc., authorized the Bonifacio Bros., Inc. to repair the car with some materials supplied by the Ayala
Autoparts Co.

ISSUE:

Whether there is privity of contract between the Bonifacio Bros. Inc and the Ayala Autoparts Co. on the one hand
and the insurance company on the other.

RULING:

No. A policy of insurance is a distinct and independent contract between the insured and insurer. A third person
has no right in law or equity to the proceeds of an insurance unless there is a contract or trust, express or implied
between the insured and the third person.

The Appellants are not mentioned in the contract as parties thereto, nor is there any clause or provision thereof
from which we can infer that there is an obligation on the part of the insurance company to pay the cost directly to
them. The clause is an insurance policy, authorizing the owner of the damaged vehicle to contract for its repairs
does not mean that the repairman is entitled to collect the cost of the repair out of the proceeds of the insurance.
It merely establishes the procedure that the insured has to follow in order to be entitled to indemnity for repair.

On the other hand, the “loss payable” clause of the insurance policy stipulates “Loss, if any, is payable to H. S.
Reyes, Inc.” indicating that it was only the H. S. Reyes Inc. which intended to be benefited. Where the mortgagee is
the beneficiary in a car insurance, it has a better right than the repairman to the insurance proceeds.
33. FGU INSURANCE CORPORATION, Petitioners,
vs.
THE COURT OF APPEALS, SAN MIGUEL CORPORATION, and ESTATE OF ANG GUI, represented by LUCIO, JULIAN,
and JAIME, all surnamed ANG, and CO TO, Respondents.

FACTS:

Anco Enterprises Company (ANCO), a partnership between Ang Gui and Co To, was engaged in the shipping
business operating two common carriers M/T ANCO tugboat D/B Lucio barge - no engine of its own, it could not
maneuver by itself and had to be towed by a tugboat for it to move from one place to another. September 23
1979: San Miguel Corporation (SMC) shipped from Mandaue City, Cebu, on board the D/B Lucio, for towage by
M/T ANCO: 25,000 cases Pale Pilsen and 350 cases Cerveza Negra - consignee SMC’s Beer Marketing Division
(BMD)-Estancia Beer Sales Office, Estancia, Iloilo 15,000 cases Pale Pilsen and 200 cases Cerveza Negra - consignee
SMC’s BMD-San Jose Beer Sales Office, San Jose, Antique September 30, 1979: D/B Lucio was towed by the M/T
ANCO arrived and M/T ANCO left the barge immediately The clouds were dark and the waves were big so SMC’s
District Sales Supervisor, Fernando Macabuag, requested ANCO’s representative to transfer the barge to a safer
place but it refused so around the midnight, the barge sunk along with 29,210 cases of Pale Pilsen and 500 cases of
Cerveza Negra totalling to P1,346,197 When SMC claimed against ANCO it stated that they agreed that it would
not be liable for any losses or damages resulting to the cargoes by reason of fortuitous event and it was agreed to
be insured with FGU for 20,000 cases or P858,500 ANCO filed against FGU FGU alleged that ANCO and SMC failed
to exercise ordinary diligence or the diligence of a good father of the family in the care and supervision of the
cargoes RTC: ANCO liable to SMC and FGU liable for 53% of the lost cargoes CA affirmed

ISSUE:

W/N FGU should be exempted from liability to ANCO for the lost cargoes because of a fortuitous event and
negligence of ANCO

RULING:

Yes. Affirmed with modification. Third-party complainant is dismissed. Art. 1733. Common carriers, from the
nature of their business and for reasons of public policy are bound to observe extraordinary diligence in the
vigilance over the goods and for the safety of the passengers transported by them, according to all the
circumstances of each case. Such extraordinary diligence in vigilance over the goods is further expressed in Articles
1734, 1735, and 1745 Nos. 5, 6, and 7. Art. 1734. Common carriers are responsible for the loss, destruction, or
deterioration of the goods, unless the same is due to any of the following causes only: (1) Flood, storm,
earthquake, lightning, or other natural disaster or calamity;
34. FINMAN GENERAL ASSURANCE CORPORATION, petitioner,
vs.
COURT OF APPEALS and USIPHIL INCORPORATED, respondents.

FACTS:

On October 22, 1986, deceased, Carlie Surposa was insured with petitioner Finman General Assurance Corporation
with his parents, spouses Julia and Carlos Surposa, and brothers Christopher, Charles, Chester and Clifton, all
surnamed, Surposa, as beneficiaries. While said insurance policy was in full force and effect, the insured, Carlie
Surposa, died on October 18, 1988 as a result of a stab wound inflicted by one of the three (3) unidentified men.
Private respondent and the other beneficiaries of said insurance policy filed a written notice of claim with the
petitioner insurance company which denied said claim contending that murder and assault are not within the
scope of the coverage of the insurance policy. Private respondent filed a complaint with the Insurance Commission
which rendered a favorable response for the respondent. The appellate court ruled likewise. Petitioner filed this
petition alleging grave abuse of discretion on the part of the appellate court in applying the principle of "expresso
unius exclusio alterius" in a personal accident insurance policy, since death resulting from murder and/or assault
are impliedly excluded in said insurance policy considering that the cause of death of the insured was not
accidental but rather a deliberate and intentional act of the assailant. Therefore, said death was committed with
deliberate intent which, by the very nature of a personal accident insurance policy, cannot be indemnified.

ISSUE:

Whether or not the insurer is liable for the payment of the insurance premiums

RULING:

Yes, the insurer is still liable. Contracts of insurance are to be construed liberally in favor of the insured and strictly
against the insurer. Thus ambiguity in the words of an insurance contract should be interpreted in favor of its
beneficiary. The terms "accident" and "accidental" as used in insurance contracts have not acquired any technical
meaning, and are construed by the courts in their ordinary and common acceptation. Thus, the terms have been
taken to mean that which happen by chance or fortuitously, without intention and design, and which is
unexpected, unusual, and unforeseen. Where the death or injury is not the natural or probable result of the
insured's voluntary act, or if something unforeseen occurs in the doing of the act which produces the injury, the
resulting death is within the protection of the policies insuring against death or injury from accident. In the case at
bar, it cannot be pretended that Carlie Surposa died in the course of an assault or murder as a result of his
voluntary act considering the very nature of these crimes. Neither can it be said that where was a capricious desire
on the part of the accused to expose his life to danger considering that he was just going home after attending a
festival. Furthermore, the personal accident insurance policy involved herein specifically enumerated only ten (10)
circumstances wherein no liability attaches to petitioner insurance company for any injury, disability or loss
suffered by the insured as a result of any of the stimulated causes. The principle of " expresso unius exclusio
alterius" — the mention of one thing implies the exclusion of another thing — is therefore applicable in the instant
case since murder and assault, not having been expressly included in the enumeration of the circumstances that
would negate liability in said insurance policy cannot be considered by implication to discharge the petitioner
insurance company from liability for, any injury, disability or loss suffered by the insured. Thus, the failure of the
petitioner insurance company to include death resulting from murder or assault among the prohibited risks leads
inevitably to the conclusion that it did not intend to limit or exempt itself from liability for such death.
35. E. M. BACHRACH, plaintiff-appellee,
vs.
BRITISH AMERICAN ASSURANCE COMPANY, a corporation, defendant-appellant.

FACTS:

Bachrach insured properties of its general furniture shop with British. The properties were subsequently destroyed
by fire. Bachrach claims from the insurance company. The claim was denied on the ff grounds: The policy was
allegedly forfeited because the insured stored varnishes and paints within the premises; Insured stored gasoline in
the building; and Bachrach executed a chattel mortgage on the properties insured without the consent of the
insured.

ISSUE:

Whether or not Bachrach can claim the proceeds of the policy.

RULING:

Yes. The policy was NOT forfeited due to the strong paints and varnishes. There was no express provision
pertaining to it and these paints and varnishes are incidental to the business of the insured to keep the furniture in
a saleable condition. The gasoline stored within the premises was in the reservoir of the car and thus does not
violate any provision in the policy. There is no express prohibition against the execution of a chattel mortgage on
the property insured.
36. JACQUELINE JIMENEZ VDA. DE GABRIEL, petitioner,
vs.
HON. COURT OF APPEALS and FORTUNE INSURANCE & SURETY COMPANY, INC., respondents.

FACTS:

The petitioner for review on certiorari seeks the reversal of the decision of the Court of Appeals setting aside the
judgment of the Regional Trial Court which ordered private respondent Fortune Insurance and Surety Company,
Inc., to pay petitioner Jacqueline Jimenez Vda. de Gabriel, the surviving spouse and beneficiary in an accident
(group) insurance of her dead husband, the amount of ₱100,000.00, plus legal interest.

Marcelino Gabriel, the insured, was employed by Emerald Construction & Development Corporation (“ECDC”) at its
construction project in Iraq. He was covered by a personal accident insurance in the amount of ₱100,000.00 under
a group policy procured from private respondent by ECDC for its overseas workers. On May 22, 1982, within the
life of the policy, Gabriel died in Iraq. A year later, or on July 12, 1983, ECDC reported Gabriel’s death to private
respondent by telephone. Ultimately private respondent denied the claim of ECDC on the ground of prescription.
Petitioner went to court alleging that her husband died of electrocution while working.

ISSUE:

Whether or not the cause of action had prescribed.

RULING:

Yes. On the issue of “prescription”, private respondent correctly invoked Section 384 of the Insurance Code; viz:
“Sec. 384. Any person having any claim upon the policy issued pursuant to this chapter shall without any
unnecessary delay, present to the insurance company concerned a written notice of claim setting forth the nature,
extent and duration of the injuries sustained as certified by a duly licensed physician. Notice of claim must be filed
within six months from date of the accident, otherwise, the claim shall be deemed waived. Action or suit for
recovery of damage due to loss or injury must be brought, in proper cases, with the Commissioner on the Courts
within one year from the denial of the claim, otherwise, the claimant’s right of action shall prescribe.” The notice
of death was given to private respondent, concededly, more than a year after the death of petitioner’s husband.
Private respondent in invoking prescription, was not referring to the one-year period from the denial of the claim
within which to file an action against an insurer but obviously to the written notice of claim that had to be
submitted within six months from the time of the accident.
37. TAN IT, plaintiff-appellant,
vs.
SUN INSURANCE OFFICE, defendant-appellant.

FACTS:

On November 25, 1924, the Sun Insurance Office issued to the Chinese merchant, Tan It, a policy of fire insurance
covering certain goods and merchandise then deposited in the bodega situated at Nos. 326-240 Calle Nueva,
Binondo, Manila. The policy was good for one year. It stipulated that in case of fire the insurer was to pay the
insured three-fourths of the value of the goods, but in no case exceeding P30,000. The policy contained other
clauses, particularly one relating to fraudulent claims.

On November 1, 1925, a fire of unknown origin, destroyed a portion of the goods and merchandise covered by the
insurance policy. On November 3, 1925, Tan It presented a verified claim of the alleged loss suffered by him on
account of the fire. On November 5, 1925, the representatives of the insurance company, consisting of Messrs.
Arthur Ruh from Kuenzle & Streiff, Inc., the agency of the insurance company; James C. Glegg and Carlos Lintag
from Hunter Bayne & Co., fire adjusters; and D. J. Awad and M. Goldstein, merchants, proceeded to the scene of
the fire , and in the presence and with the assistance of Tan It, made a physical inventory of the pieces of
merchandise existing in the bodega. Subsequently, the salvaged merchandise was sold for P3,000, which was
deposited in a bank on behalf of whom it may concern. The parties having found it impossible to arrive at an
amicable settlement, the instant suit is the result.

ISSUE:

Whether or not Tan It's claim was merely erroneous and exaggerated as found by the trial court, or fraudulent and
thus voidable as contended by the insurance company.

RULING:

Clause 13 of the contract of insurance provides that "If the claim be in any respect fraudulent, or if any false
declaration be made or used in support thereofall benefit under this Policy shall be forfeited." With this clause in
the policy to the forefront, a few figures should be set down to elucidate the situation. Plaintiff's verified claim
totalled P31,860.85, of which, in accordance with the terms of the policy, three-fourths was asked, or P23,895.64.
Dependant's inventory of the goods found after the fire came to P13,113. The difference between plaintiff's claim
and defendant's estimate of the loss, which was confirmed in the trial court, was P18,747.85. In connection with
these figures plaintiff suggests too low a valuation by the representatives of the defendant. Computed at plaintiff's
valuation, the goods inventoried by the defendant's committee would amount to P19,346.30. There would,
however, still remain a considerable void between the two amounts, of P12.514.55. Plaintiff's additional effort to
account for discrepancy between the two inventories by endeavoring to show that certain goods were not
inventoried by defendant and that other goods were completely burned, has not been successful.
38. NORMAN NODA, petitioner,
vs.
HONORABLE GREGORIA CRUZ-ARNALDO, in her capacity as Insurance Commissioner, and ZENITH INSURANCE
CORPORATION, respondents.

FACTS:

Norman R. Noda obtained from respondent Zenith Insurance Corporation two fire insurance policies. While both
policies were in force, fire destroyed petitioner's insured properties at the market site and at Barreda St. When
petitioner failed to obtain indemnity on his claims from respondent Zenith, he filed a complaint with the Insurance
Commission praying that respondent company be ordered to pay him "the sum of P130,000 representing the value
of the two [2] policies insured by respondent with interest at 12% per annum, plus damages, attorney's fees and
other expenses of litigation. In its answer Zenith interposed that petitioner had no cause of action; that Policy No.
F-03724 was not in full force and effect at the time of the fire because the premium on the policy was not paid;
that Zenith's liability under Policy No. F-03734, if any, was limited to P15,472.50 in view of the coinsurance; and
that petitioner failed to substantiate his claim as to the value of the goods reputedly destroyed by fire and
consequently, Zenith could not be held answerable for the same. Insurance Commissioner did not allowed Noda to
recover under said policy and the actual, moral and exemplary damages prayed for.

ISSUE:

Whether or not Zenith Insurance Corporation is liable.

RULING:

Yes. We find that respondent Commissioner acted with grave abuse of discretion when she denied petitioner's
claim for indemnity under Policy No. F-03734 because of what she perceived as insufficient proof. To prove the
existence of the stocks in trade covered by Policy No. F-03734, petitioner offered his testimony and that of his wife
as well as documentary exhibits. The foregoing evidence for petitioner preponderantly showed the presence of
some P590,000 worth of goods in his retail store during the fire of November 9, 1977.The report even took into
account the appraisals of the other adjusters and concluded that the total loss sustained by petitioner in his
household effectsandstocks in trade reached P379,302.12. But after apportioning said amount among petitioner's
six different in surers [the co-insurance being known to Zenith], the liability of Zenith was placed at P60,592.10. It
therefore recommended that Zenith pay the petitioner the amount of P60, 592.10. While the insurer and the
Insurance Commissioner for that matter, have the right to reject proofs of loss if they are unsatisfactory, they may
not set up for themselves an arbitrary standard of satisfaction. Substantial compliance with the requirements will
always be deemed sufficient. The denial of petitioner's demand for exemplary damages by respondent
Commissioner must, however, be sustained. There is no showing that Zenith, in contesting payment, had acted in a
wanton, oppressive or malevolent manner to warrant the imposition of corrective damages.
39. SALUD VILLANUEVA VDA. DE BATACLAN and the minors NORMA, LUZVIMINDA, ELENITA, OSCAR and
ALFREDO BATACLAN, represented by their Natural guardian, SALUD VILLANUEVA VDA. DE BATACLAN, plaintiffs-
appellants,
vs.
MARIANO MEDINA, defendant-appellant.

FACTS:

Shortly after midnight, on


September 13, 1952, bus No. 30 of
the Medina Transportation,
operated by
Mariano Medina left Amadeo,
Cavite, en route to Pasay City,
driven by Conrado Saylon. There
were
about 18 passengers, including the
driver and conductor. Among the
passengers were Juan Bataclán,
Felipe Lara, another passenger
whom the witnesses just called
Visaya, and a woman named
Natalia
Villanueva.
At about 2:00 in the morning,
while the bus was within of Imus,
Cavite, one of the front tires burst
and
the vehicle began to zig-zag until it
fell into a canal or ditch on the
right side of the road and turned
turtle. Some of the passengers
managed to leave the bus the best
way they could, others had to be
helped or pulled out, while the
three passengers seated beside the
driver, named Bataclán, Lara and
the Visayan and the woman
behind them named Natalia
Villanueva, could not get out
of the
overturned bus.
Some of the passengers, after they
had clambered up to the road,
heard groans and moans from
inside
the bus, particularly, shouts for
help from Bataclán and Lara, who
said that they could not get out of
the bus. There were no attempts
made to pull out or extricate and
rescue the four passengers trapped
inside the vehicle, but calls or
shouts for help were made to the
houses in the neighborhood
Shortly after midnight, on September 13, 1952, bus No. 30 of the Medina Transportation, operated byMariano
Medina left Amadeo, Cavite, en route to Pasay City, driven by Conrado Saylon. There wereabout 18 passengers,
including the driver and conductor. Among the passengers were Juan Bataclán,Felipe Lara, another passenger
whom the witnesses just called Visaya, and a woman named NataliaVillanueva. At about 2:00 in the morning, while
the bus was within of Imus, Cavite, one of the front tires burst andthe vehicle began to zig-zag until it fell into a
canal or ditch on the right side of the road and turnedturtle. Some of the passengers managed to leave the bus the
best way they could, others had to behelped or pulled out, while the three passengers seated beside the driver,
named Bataclán, Lara andthe Visayan and the woman behind them named Natalia Villanueva, could
not get out of theoverturned bus. Some of the passengers, after they had clambered up to the road, heard
groans and moans from insidethe bus, particularly, shouts for help from Bataclán and Lara, who said that they
could not get out ofthe bus. There were no attempts made to pull out or extricate and rescue the four passengers
trappedinside the vehicle, but calls or shouts for help were made to the houses in the neighborhood

ISSUE:

Whether or Not Medina Transportation is liable for the death of Juan Bataclán

RULING:

YES, Medina Transportation is liable for the death of Juan Bataclán.

The Court held that the proximate


cause was the overturning of the bus
because when the vehicle turned
not only on its side but completely
on its back, the leaking of the
gasoline from the tank was not
unnatural or unexpected.
The Court held that the proximate cause was the overturning of the bus because when the vehicle turnednot only
on its side but completely on its back, the leaking of the gasoline from the tank was notunnatural or unexpected.
The coming of the men with a lighted torch was in response to the call for help, made not only by thepassengers,
but most probably, by the driver and the conductor themselves, and that because it was dark(about 2:30 in the
morning), the rescuers had to carry a light with them, and coming as they did from arural area where lanterns and
flashlights were not available.

CHAPTER 8

40. MANILA MAHOGANY MANUFACTURING CORPORATION, petitioner,


vs.
COURT OF APPEALS AND ZENITH INSURANCE CORPORATION, respondents.

FACTS:

Petitioner insured its Mercedes Benz 4-door sedan with respondent insurance company . The insured vehicle was
bumped and damaged by a truck owned by San Miguel Corporation (SMC). For the damage caused, respondent
company paid petitioner ₱ 5,000.00 in amicable settlement. Petitioner’s general manager executed a Release of
Claim, subrogating respondent company to all its right to action against San Miguel Corp. Respondent company
wrote the Insurer Adjusters, Inc. to demand reimbursements from San Miguel Corporation of the amount it had
paid petitioner. Insurer Adjusters, Inc. refused reimbursement alleging that SMC had already paid petitioner ₱
4,500.00 for the damages to petitioner’s motor vehicle, as evidenced by a cash voucher and Release of Claim
executed by the General Manager of petitioner discharging SMC from “ all actions, claims, demands the right of
action that now exist or hereafter develop arising out of or as a consequence of the accident.
Respondent demanded the ₱ 4,500.00 amount from petitioner. Petitioner refused. Suit was filed for recovery. City
Court ordered petitioner to pay respondent. CFI affirmed. CA affirmed with modification that petitioner was to pay
respondent the total amount of ₱ 5,000.00 it had received from respondent. Petitioner’s argument: Since the total
damages were valued at P9,486.43 and only ₱ 5,000.00 was received by petitioner from respondent, petitioner
argues that it was entitled to go after SMC to claim the additional which was eventually paid to it. Respondent’s
argument: No qualification to its right of subrogation.

ISSUE:

Whether or not the insured should pay the insurer despite that the subrogation in the Release of Claim was
conditioned on recovery of the total amount of damages that the insured has sustained.

RULING:

NO. Supreme Court said there being no other evidence to support its allegation that a gentleman’s agreement
existed between the parties, not embodied in the Release of Claim, such Release of Claim must be taken as the
best evidence of the intent and purpose of the parties. CA was correct in holding petitioner should reimburse
respondent ₱ 5,000.00.

When Manila Mahogany executed another release claim discharging SMC from all rights of action after the insurer
had paid the proceeds of the policy – the compromise agreement of ₱ 5,000.00– the insurer is entitled to recover
from the insured the amount of insurance money paid. Petitioner by its own acts released SMC, thereby defeating
respondent’s right of subrogation, the right of action against the insurer was also nullified. Since the insurer can be
subrogated to only such rights as the insured may have, should the insured, after receiving payment from the
insurer, release the wrongdoer who caused the loss, the insurer losses his rights against the latter. But in such a
case, the insurer will be entitled to recover from the insured whatever it has paid to the latter, unless the release
was made with the consent of the insurer.

41. FEDERAL EXPRESS CORPORATION, petitioner,


vs.
AMERICAN HOME ASSURANCE COMPANY and PHILAM INSURANCE COMPANY, INC., respondents.

FACTS:

Shipper SMITHKLINE USA delivered to carrier Burlington Air Express, an agent of herein petitioner, a cargo
shipment, insured with respondent which consist of 109 cartons of veterinary biological for delivery to consignee
SMITHKLINE and French Overseas Company in Makati City with the words, “REFRIGERATE WHEN NOT IN TRANSIT”
and “PERISHABLE” stamp marked on its face. However, 12 days after the cargoes arrived in Manila, it was found
out that the same were stored only in a room with 2 air conditioners running in the warehouse of Cargohaus Inc.,
to cool the place instead of a refrigerator.

As a consequence of the result of the veterinary biological test, SMITHKLINE abandoned the shipment and,
declaring “total loss” for the unusable shipment, filed a claim with AHAC through its representative in the
Philippines, The Philam Insurance Co., Inc., (PHILAM) which recompensed SMITHKLINE for the whole insured
amount. Thereafter, PHILAM filed an action for damages against FEDEX imputing negligence on either or both of
them in the handling of the cargo where it was decided that FEDEX is solidarily liable with Cargohaus Inc.
ISSUE:

Whether or not FEDEX is liable for damage to or loss of the insured goods?

RULING:

No. Upon receipt of the insurance proceeds, the consignee (SMITHKLINE) executed a subrogation receipt in favor
of respondents authorizing them “to file claims and begin suit against any such carrier, person, vessel, corporation
or government.” Undeniably, the consignee had a legal right to receive the goods in the same condition it was
delivered for transport to petitioner and if that right was violated, the consignee would have a cause of action
against the person responsible therefor.

In the exercise of its subrogatory right, an insurer may proceed against an erring carrier and to all intents and
purposes, it stands in the place and in substitution of the consignee.

43. FILIPINO MERCHANTS INSURANCE COMPANY, INC., petitioner,


vs.
HONORABLE JOSE ALEJANDRO, Presiding Judge of Branch XXVI of the Court of First Instance of Manila
and FROTA OCEANICA BRASILIERA, respondents.

FACTS:

Petitioner insured the goods of Plaintiff Mr. Choa Seng. The latter filed a a complaint against petitioner before the
then Court of First Instance of Manila for recovery of a sum of money under the marine insurance policy on cargo
alleging that the goods insured with the petitioner sustained loss and damage. SS Frotario which was owned and
operated by private respondent Frota Oceanica Brasiliera, (Frota) discharged the goods at the port of Manila on
December 13, 1976. The said goods were delivered to the arrastre operator E. Razon, Inc., on December 17, 1976
and on the same date were received by the consignee-plaintiff. An identical case was likewise files by Joseph Chua
whose goods were received by the consignee on January 25-28, 1977. Petitioner filed a third-party complaint
against the carrier, private respondent Frota and the arrastre contractor, E. Razon, Inc. for indemnity, subrogation,
or reimbursement in the event that it is held liable to the plaintiff. It filed an amended third-party complaint
against respondent carrier, the Australia-West Pacific Line (Australia-West). The private respondents alleged in
their separate answers that the petitioner is already barred from filing a claim because under the Carriage of
Goods by Sea Act, the suit against the carrier must be filed within one year after delivery of the goods or the date
when the goods should have been delivered. The petitioner contended that the provision relied upon by the
respondents applies only to the shipper and not to the insurer of the goods. Respondent judge dismissed the
complaints filed on the ground that the action has already prescribed.

ISSUE:

Whether the one-year prescriptive period within which to file a case against the carrier also applies to a claim filed
by an insurer who stands as a subrogee to the insured.

RULING:

Yes. Section 3(b) of the Carriage of Goods by Sea Act provides in part: "In any event the carrier and the ship shall
be discharged from all liability in respect of loss or damage unless suit is brought within one year after delivery of
the goods or the date when the goods should have been delivered: Provided, that if a notice of loss or damage,
either apparent or concealed, is not given as provided for in this section, that fact shall not affect or prejudice the
right of the shipper to bring the suit within one year after the delivery of the goods or the date when the goods
should have been delivered.” Clearly, the coverage of the Act includes the insurer of the goods. Otherwise, what
the Act intends to prohibit after the lapse of the one-year prescriptive period can be done indirectly by the shipper
or owner of the goods by simply filing a claim against the insurer even after the lapse of one year. This would be
the result if we follow the petitioner's argument that the insurer can, at any time, proceed against the carrier and
the ship since it is not bound by the time-bar provision. In this situation, the one-year limitation will be practically
useless. This could not have been the intention of the law which has also for its purpose the protection of the
carrier and the ship from fraudulent claims by having "matters affecting transportation of goods by sea be decided
in as short a time as possible" and by avoiding incidents which would "unnecessarily extend the period and permit
delays in the settlement of questions affecting the transportation."

45. ABOITIZ SHIPPING CORPORATION, petitioner,


vs.
INSURANCE COMPANY OF NORTH AMERICA, respondent.

FACTS:

MSAS Cargo International Limited and/or Associated and/or Subsidiary Companies (MSAS) procured an “all-risk”
marine insurance policy from ICNA UK Limited of London for its cargo, consisting of wooden work tools and
workbenches purchased by consignee Science Teaching Improvement Project (STIP) which was later on received by
Aboitiz and shipped to Cebu. However upon arrival, the checker noted that the crates were slightly broken or
cracked at the bottom causing the cargo to be withdrawn by the representative of the consignee, STIP and
delivered to Don Bosco Technical High School, Punta Princesa, Cebu City where it was received by Mr. Bernhard
Willing who later on reported the damage to Aboitiz

Consignee filed a claim against ICNA who then paid consignee and a subrogation receipt was duly signed by Willig.
ICNA then advised Aboitiz of the receipt signed in its favor but received no reply so it filed for collection at the RTC.
ISSUE:

Whether or not ICNA can claim under the right of subrogation.

RULING:

Yes. Only when that corporation is “transacting” or “doing business” in the country with a license necessary before
it can institute suits. It may, however, bring suits on isolated business transactions, which is not prohibited under
Philippine Law. The policy benefits any subsequent assignee, or holder, including the consignee, who may file
claims on behalf of the assured.

47. PAN MALAYAN INSURANCE CORPORATION, petitioner,


vs.
COURT OF APPEALS, ERLINDA FABIE AND HER UNKNOWN DRIVER, respondents.

FACTS:

Pan Malayan filed a complaint for damages with the RTC of Makati against private respondents Erlinda Fabie and
her driver. It insured a Mitsubishi Colt Lancer car registered in the name of Canlubang. Due to the carelessness,
recklessness, and imprudence of the unknown driver of a pick-up, the insured car was hit and suffered damages in
the amount of P42,052.00.

Pan Malayan defrayed the cost of repair of the insured car, and therefore was subrogated to the rights of
Canlubang against the driver of the pick-up and his employer, Erlinda Fabie. Despite repeated demands,
defendants failed and refused to pay the claim of Pan Malay. Defendants/Private Respondents alleged that Pan
Malayan had no cause of action against them because payment under the “own damage” clause of the insurance
policy precluded subrogation under Article 2207 of the Civil Code, since indemnification thereunder was made on
the assumption that there was no wrongdoer or no third party at fault. RTC dismissed the case for no cause of
action and denied its motion for reconsideration. The CA affirmed the trial courts decision.

ISSUE:

Whether or not the insurer may institute ac action to recover the amount it had paid its assured in settlement of
an insurance claim against private respondents.

RULING:

Pan Malayan is correct. If the insured property is destroyed or damaged through the fault or negligence of a party
other than the assured, then the insurer, upon payment to the assured, will be subrogated to the rights of the
assured to recover from the wrongdoer to the extent that the insurer has been obligated to pay. Payment by the
insurer to the assured operates as an equitable assignment to the former of all remedies, which the latter ma have
against the third party whose negligence or wrongful act caused the loss.

The right of subrogation is not dependent upon, nor does it grow out of any privity of contract or upon written
assignment of claim. It accrues simply upon payment of the insurance claim by the insurer.

49. COMPAÑIA MARITIMA, petitioner,


vs.
INSURANCE COMPANY OF NORTH AMERICA, respondent.

FACTS:

Macleod and Company of the Philippines contracted by telephone the services of the Compañia Maritima, a
shipping corporation,for the shipment of 2,645 bales of hemp from the former's Sasa private pier at Davao City to
Manila and for their subsequent transhipment to Boston, Massachusetts, U.S.A. on board the S.S. Steel Navigato.
Petitioner sent to Macleod's private wharf LCT Nos. 1023 and 1025 which was manned by a patron to Get the
cargo. They issued a receipt which show that they have transported the cargo from Davao to Manila for further
transshipment to U,S,A. One of the barge was sank resulting in the loss and damages of its cargo. Macloud filed its
claim of insurance which was paid by the insurance company. Subrogated in the right of Macloud, the insurance
company demand from the petitioner its liability but the latter refused to comply. The insurance company filed a
complaint against the petitioner before the trial court who order the petitioner to pay the plaintiff insurance
company and this was affirmed by the CA.
ISSUE:

Can the insurance company maintain this suit without proof of its personality to do so?

RULING:

According to the Supreme Court, since the contract between the shipper and the carrier in this case was valid and
properly established, when macloud suffered damages because of the doing of petitioner, the insurer who pay the
insurance claim was subrogated on behalf of Macloud giving the insurance company a personality to institute this
case.

50. MALAYAN INSURANCE CO., INC., petitioner,


vs.
THE HON. COURT OF APPEALS (THIRD DIVISION) MARTIN C. VALLEJOS, SIO CHOY, SAN LEON RICE
MILL, INC. and PANGASINAN TRANSPORTATION CO., INC., respondents.

FACTS:

June 7, 1981: Malayan insurance co., inc. (MICO) issued to Coronacion Pinca, Fire Insurance Policy for her property
effective July 22, 1981, until July 22, 1982. On October 15,1981, MICO allegedly cancelled the policy for non-
payment, of the premium and sent the corresponding notice to Pinca. On December 24, 1981, payment of the
premium for Pinca was received by Domingo Adora, agent of MICO. On January 15, 1982, Adora remitted this
payment to MICO,together with other payments. On January 18, 1982, Pinca’s property was completely burned.
On February 5, 1982, Pinca’s payment was returned by MICO to Adora on the ground that her policy had been
cancelled earlier but Adora refused to accept it and instead demanded for payment. Under Section 416 of the
Insurance Code, the period for appeal is thirty days from notice of the decision of the Insurance Commission. The
petitioner filed its motion for reconsideration on April 25, 1981, or fifteen days such notice, and the reglementary
period began to run again after June 13, 1981, date of its receipt of notice of the denial of the said motion for
reconsideration. As the herein petition was filed on July 2, 1981, or nineteen days later, there is no question that it
is tardy by four days. Insurance Commission, favored Pinca MICO

ISSUE:

W/N MICO should be liable because its agent Adora was authorized to receive it

RULING:

YES. petition is DENIED SEC. 77. An insurer is entitled to payment of the premium as soon as the thing is exposed to
the peril insured against. Notwithstanding any agreement to the contrary, no policy or contract of insurance issued
by an insurance company is valid and binding unless and until the premium thereof has been paid, except in the
case of a life or an industrial life policy whenever the grace period provision applies.
SEC. 306.

Any insurance company which delivers to an insurance agant or insurance broker a policy or contract of insurance
shall be demmed to have authorized such agent or broker to receive on its behalf payment of any premium which
is due on such policy or contract of insurance at the time of its issuance or delivery or which becomes due thereon.
Payment to an agent having authority to receive or collect payment is equivalent to payment to the principal
himself; such payment is complete when the money delivered is into the agent’s hands and is a discharge of the
indebtedness owing to the principal.

51. COASTWISE LIGHTERAGE CORPORATION, petitioner,


vs.
COURT OF APPEALS and the PHILIPPINE GENERAL INSURANCE COMPANY, respondents.

FACTS:

The consignee entered into a contract of affreightment which is to transport molasses from the province of Negros
to Manila with the carrier using the latter’s barges. The barges were towed in tandem by the tugboat MT Marica,
also owned by the carrier. While approaching the pier of destination, one of the barges, “Coastwise 9” was struck
and as a result, the molasses at the cargo tanks were contaminated and rendered unfit for the use it was intended.
The consignee rejected the shipment of molasses as a total loss. The insurer paid the consignee the amount
representing the value of the damaged cargo of molasses.

ISSUE:
WON the insurer was subrogated into the rights of the consignee against the carrier, upon payment by the insurer
of the value of the consignee’s goods lost while on board one of the carrier’s vessels.

RULING:

Petitioner carrier was liable for breach of the contract of carriage it entered into with the consignee. In accordance
with Art. 2207, payment by the insurer to the assured operated as an equitable assignment to the former of all
remedies which the latter may have against the third party whose negligence or wrongful act caused the loss. If the
insured property is destroyed or damaged through the fault or negligence of a party other than the assured, then
the insurer, upon payment to the assured will be subrogated to the rights of the assured to recover from the
wrongdoer to the extent that the insurer has been obligated to pay.

52. THE PHILIPPINE AMERICAN GENERAL INSURANCE COMPANY, INC., petitioner,


vs.
COURT OF APPEALS and FELMAN SHIPPING LINES, respondents.

FACTS:

On 6 July 1983 Coca-Cola Bottlers


Philippines,
Inc., loaded on board "MV Asilda," a
vessel owned and
operated by respondent Felman
Shipping Lines, 7,500
cases of 1-liter Coca-Cola softdrink
bottles to be
transported from Zamboanga City to
Cebu City. The
shipment was insured with petitioner
Philippine American
General Insurance Co., Inc., under
Marine Open. On 7
July 1983, the vessel sank in the waters of
Zamboanga
del Norte bringing down her entire cargo
with her including
the subject 7,500 cases of 1-liter
Coca-Cola softdrink
bottles.
on 29 November 1983 PHILAMGEN
sued the
shipowner for sum of money and
damages. On 15
February 1985 FELMAN filed a motion to
dismiss based
on the affirmative defense that FELMAN
had abandoned
all its rights, interests and ownership
over "MV Asilda"
together with her freight and
appurtenances for the
purpose of limiting and extinguishing its
liability under Art.
587 of the Code of Commerce.
On 6 July 1983 Coca-Cola Bottlers Philippines,Inc., loaded on board "MV Asilda," a vessel owned andoperated by
respondent Felman Shipping Lines, 7,500cases of 1-liter Coca-Cola softdrink bottles to betransported
from Zamboanga City to Cebu City. Theshipment was insured with petitioner Philippine AmericanGeneral
Insurance Co., Inc., under Marine Open. On 7July 1983, the vessel sank in the waters of Zamboangadel Norte
bringing down her entire cargo with her includingthe subject 7,500 cases of 1-liter Coca-Cola
softdrinkbottles.on 29 November 1983 PHILAMGEN sued theshipowner for sum of money and
damages. On 15February 1985 FELMAN filed a motion to dismiss basedon the affirmative defense that FELMAN
had abandonedall its rights, interests and ownership over "MV Asilda"together with her freight and
appurtenances for thepurpose of limiting and extinguishing its liability under Art.587 of the Code of Commerce.

ISSUE:

WON the limited liability under Art.


587 of the
Code of Commerce should apply
WON the limited liability under Art. 587 of theCode of Commerce should apply

RULING:

Art. 587 of the Code of Commerce is NOTapplicable to the case at bar. The ship agent is liable forthe
negligent acts of the captain in the care of goodsloaded on the vessel. This liability however can be
limitedthrough abandonment of the vessel, its equipment andfreightage as provided in Art. 587.
Nonetheless, there areexceptional circumstances wherein the ship agent couldstill be held answerable
despite the abandonment, aswhere the loss or injury was due to the fault of theshipowner and the
captain. The international rule is to theeffect that the right of abandonment of vessels, as a legallimitation of a
shipowner's liability, does not apply to caseswhere the injury or average was occasioned by
theshipowner's own fault. It must be stressed at this pointthat Art. 587 speaks only of situations where the
fault ornegligence is committed solely by the captain. Where theshipowner is likewise to be blamed, Art.
587 will notapply. It was already established at the outset that thesinking of "MV Asilda" was due to its
unseaworthinesseven at the time of its departure from the port ofZamboanga. Closer supervision on
the part of theshipowner could have prevented this fatal miscalculation.As such, FELMAN was equally
negligent. It cannottherefore escape liability through the expedient of filing anotice of abandonment of the
vessel by virtue of Art. 587of the Code of Commerce.

53. KEPPEL CEBU SHIPYARD, INC., Petitioner,


vs.
PIONEER INSURANCE AND SURETY CORPORATION, Respondent.

FACTS:

Keppel Cebu Shipyard, Inc. (KCSI) and WG&A Jebsens Shipmanagement, Inc. (WG&A) executed a Shiprepair
Agreement wherein KCSI would renovate and reconstruct WG&A’s M/V “Superferry 3” using its dry docking
facilities pursuant to its restrictive safety and security rules and regulations. Prior to this, the vessel was also
insured with Pioneer for $8.47M. In the course of its repair, the vessel was gutted by fire. WG&A declared its “total
constructive loss” and filed an insurance claim with Pioneer. Pioneer paid P8.47M for which WG&A issued a Loss
and Subrogation Receipt. Pioneer then tried to collect from KCSI but the latter disclaimed responsibility refused to
pay despite repeated demands.
Thus, Pioneer filed a Request for Arbitration before the Construction Industry Arbitration Commission (CIAC).
Pioneer avers that it has been subrogated to the claims of WG&A, and the KCSI was liable given that, among
others, the proximate cause of the accident was the negligence of its employee, Sevillejo (welder) in cutting the
bulkhead door. On the other hand, KCSI disclaimed liability, imputing negligence on “Dr. Joniga’s and the Vessel’s
deliberate decision to have Sevillejo undertake cutting work in inherently dangerous conditions created by them.”
KCSI also claimed that there was no proper subrogation and that there was no “total constructive loss.”

ISSUE:

I s s u b r o g a ti o n p r o p e r ? I f p r o p e r , t o w h a t e x t e n t c a n s u b r o g a ti o n b e m a d e ?

RULING:

YES, THE SUBROGATION WAS PROPER. PIONEER MAY CLAIM THE AMOUNT OF THE LOSS, SUBJECT TO THE CLAIM
OF KCSI AS TO THE SALVAGE VALUE OF THE VESSEL.
The Loss and Subrogation Receipt issued by WG&A to Pioneer is the best evidence of payment of the insurance
proceeds to the former, and no controverting evidence was presented by KCSI to rebut the presumed authority of
the signatory to receive such payment.

Furthermore, there was “total constructive loss,” thus, Pioneer may recover the amount of loss paid to WG&A. In
the determination of whether there was “total constructive loss,” Section 139 of the Insurance Code should
governs because a) Philippine laws were deemed written in every contract; b) the subject marine insurance policies
expressly provided for such applicability.

54. RCJ BUS LINES, INCORPORATED, Petitioner,


vs.
STANDARD INSURANCE COMPANY, INCORPORATED, Respondent.

FACTS:

Standard Insurance Co., Inc. (STANDARD) filed a complaint against petitioners FLor Bola Mangoba and RCJ Bus
Lines. The complaint was predicated upon an accident which involves the Mitsubishi Lancer and the RCJ Bus Lines.
Upon seeing a pile of gravel and sand on the road, the Toyota Corolla, which is ahead of the Mitsubishi Lancer,
stopped on its tracks. The Mitsubishi Lancer followed suit and also halted. At this point, the bus hit and bumped
the rear portion of the Mitsubishi Lancer causing it to move forward and hit the Toyota Corolla in front of it. As a
result of the incident, the Mitsubishi Lancer sustained damages amounting to PhP162,151.22, representing the
cost of its repairs. Under the comprehensive insurance policy secured by Rodelene Valentino, owner of the
Mitsubishi Lancer, STANDARD reimbursed to the former the amount she expended for the repairs of her vehicle.
Rodelene then executed a Release of Claim and Subrogation Receipt, subrogating STANDARD to all rights, claims
and actions she may have against RCJ Bus Lines, Inc. and its driver, Flor Bola Mangoba. In its answer, RCJ Bus Lines,
Inc maintained, among others, that the direct, immediate and proximate cause of the accident was the negligence
of the driver of the Mitsubishi Lancer when, for no reason at all, it made a sudden stop along the National
Highway, as if to initiate and/or create an accident. The MeTC rendered its decision in favor of Standard. The RTC
affirmed with modification the MeTC’s Decision deleting the award for exemplary damages. The appellate court
found that the RTC committed no reversible error in affirming RCJ’s liability as registered owner of the bus and
employer of Mangoba

ISSUE:

Whether or Not the Court of Appeals erroneously disregarded the point that petitioner RCJ’s defense of
extraordinary diligence in the selection and supervision of its driver was made as an alternative defense;

RULING:

No. The petition has no merit. RCJ, by presenting witnesses to testify on its exercise of diligence of a good father of
a family in the selection and supervision of its bus drivers, admitted that Mangoba is its employee. Article 2180 of
the Civil Code, in relation to Article 2176, makes the employer vicariously liable for the acts of its employees. When
the employee causes damage due to his own negligence while performing his own duties, there arises the juris
tantum presumption that the employer is negligent - rebuttable only by proof of observance of the diligence of a
good father of a family. For failure to rebut such legal presumption of negligence in the selection and supervision
of employees, the employer is likewise responsible for damages, the basis of the liability being the relationship of
pater familias or on the employer’s own negligence. Mangoba, per testimony of his conductor, was ten meters
away from the Mitsubishi Lancer before the collision and was driving 60 to 75 kilometers per hour when the speed
limit was 50 kilometers per hour. The presumption under Article 2185 of the Civil Code was thus proven true:
Mangoba, as driver of the bus which collided with the Mitsubishi Lancer, was negligent since he violated a traffic
regulation at the time of the mishap. We see no reason to depart from the findings of the MeTC, RTc and appellate
court that Mangoba was negligent.

CHAPTER 9

55.
THE POLICY

CONTRACT- GEN NEED NOT TO BE WRITTEN

POLICY- MUST BE WRITTEN OR IN PRINTED FROM (CAN HAVE A BLANK SPACE)


WHY> TO PROTECT THE PUBLIC (8) FORMS

172156---malayan case

Aboitiz

Marine –not an insurance contract

G.R. No. 94071 March 31, 1992

NEW LIFE ENTERPRISES and JULIAN SY, petitioners,


vs.
HON. COURT OF APPEALS, EQUITABLE INSURANCE CORPORATION, RELIANCE SURETY AND INSURANCE CO., INC.
and WESTERN GUARANTY CORPORATION,

G.R. No. 112360               July 18, 2000

RIZAL SURETY & INSURANCE COMPANY, petitioner,


vs.
COURT OF APPEALS and TRANSWORLD KNITTING MILLS, INC.,

xxx contained and/or stored during the currency of this Policy in the premises occupied by them forming part of
the buildings situate (sic) within own Compound xxx"

Therefrom, it can be gleaned unerringly that the fire insurance policy in question did not limit its coverage to what
were stored in the four-span building. As opined by the trial court of origin, two requirements must concur in order
that the said fun and amusement machines and spare parts would be deemed protected by the fire insurance
policy under scrutiny, to wit:

"First, said properties must be contained and/or stored in the areas occupied by Transworld and second, said areas
must form part of the building described in the policy xxx" 14

'Said building of four-span lofty one storey in height with mezzanine portions is constructed of reinforced concrete
and hollow blocks and/or concrete under galvanized iron roof and occupied as hosiery mills, garment and lingerie
factory, transistor-stereo assembly plant, offices, ware house and caretaker's quarter.'

1377 OF CIVIL CODE !!!

Gulf Resorts Inc. V. Philippine Charter Insurance Corp. G.R.

-COVER NOTES

-NO SEPARATE PREMIUM (PACIFIC CASE)


"Section 60. An open policy is one in which the value of the thing insured is not agreed upon, and the amount of
the insurance merely represents the insurer’s maximum liability. The value of such thing insured shall be
ascertained at the time of the loss.

"Section 61. A valued policy is one which expresses on its face an agreement that the thing insured shall be valued
at a specific sum.

"Section 62. A running policy is one which contemplates successive insurances, and which provides that the object
of the policy may be from time to time defined, especially as to the subjects of insurance, by additional statements
or indorsements.1âwphi1

OPEN POLICY

CASE:

. TAN CHUCO, plaintiff, appellant-appellee,

vs.

YORKSHIRE FIRE AND LIFE INSURANCE COMPANY, G.R. No. L-5069

You might also like